You are on page 1of 40

Pediatrics

1) Theme: ABDOMINAL PAIN


A. Peritonitis
B. Appendicitis
C. Constipation
D. Urinary tract infection
E. Mesenteric lymphadenitis

F. Migraine
G. Intussusception
H. Henoch-Schonlein vasculitis
I. Psychosomatic
J. Shigella dysentry

For the following patients with abdominal pain, choose the most likely diagnosis.
1) A previously well 10 month old baby presenting with
G. Intussusception
drawing up of her legs with crying and some bloody runny
stools.
Note:
This is the right age group for intussception. It is often preceded by an URTI and it is thought that
mesenteric lymphadenopathy may precipitate intussception. A red currant jelly-like stool is a late sign. In
the older child, the cause of intussception may be HSP or lymphoma.
2) A 4-year-old child, with poor eating habits, iron deficiency
C. Constipation
anaemia and intermittent colicky abdominal pain.
Note:
Constipation is common in children and should be managed aggressively to establish regular bowel habit
quickly.
3) A 2-year-old girl with nephrotic syndrome who has
A. Peritonitis
developed a fever and abdominal pain.
Note:
The causes of abdominal pain in nephrotic syndrome are peritonitis (usually pneumococcal), renal vein
thrombosis and hypovolaemia.
2) Theme: Skin lesions
A. Caf au lait spots
B. Capillary haemangioma
C. Port-wine stain
D. Shagreen patch
E. Adenoma sebaceum

F. Cutis Marmorata
G. Herald's patch
H. Peri-oral pigmentation
I. Hypo pigmentation
J. Tuber

Select one option from the list above that is most suitable for the following patients
1) An 11 year old boy with learning difficulties has an acne type
E. Adenoma sebaceum
rash over the nose
Note:
relates to the condition known as Tuberous sclerosis. It is a condition where by hamartomatous lesions
become evident in early childhood. Seizures and learning difficulties are common and skin manifestations
include fibrous angiomatous lesions in the naso labial folds known as adenoma sebaceum. White macules
are seen over the trunk known as Ash leaf macules and Shagreen patches resemble goose like flesh.
Subungual fibromata are also common.
2) A 4 year old boy with a history of focal seizures and a birth
C. Port-wine stain
mark
Note:
relates to Sturge-Weber syndrome. The association and localization of aberrant vasculature in the facial
skin, eyes and meninges. Most commonly in the trigeminal nerve distribution.
3) An infant with Down's syndrome has mottled appearance to
F. Cutis Marmorata
the skin.
Note:
relates to Cutis Marmorata, a mottled, marbled type appearance to the skin. Common in Down's
syndrome as well as other Trisomies, hypothyroidism and Cornelia-de-Lange syndrome.

Pediatrics

3) Theme: Poisoning
A. Ant psychotics
B. Benzodiazepines
C. Ethylene glycol
D. Methanol
E. Opiates

F. Organophosphates
G. Salicylates
H. Sympathomimetic
I. Tricyclic antidepressants
J. Volatile solvents

Select the most appropriate agent from the above list that would elicit the following toxic effects:
1) Coma, pinpoint pupils, hyperventilation
E. Opiates
2) Hyper salivation, broncho-rrhoea, broncho- spasms,
perspiration, neuromuscular paralysis

F. Organophosphates

3) Tachypnoea, metabolic acidosis, Haematemesis, Reyes


Syndrome.

G. Salicylates

Comments:
A pinpoint pupil suggests : opiates (Heroin Morphine codeine). The hyper salivation, tachypnoea
and sweating suggest :Organophosphate poisoning. This drug inhibits ant-cholinesterase activity
promoting Ach effects. Treatment consists of anticholinergics < (Atropine dose i.v 0.05 mg/kg repeated
every 5-10 min as needed adverse effect tachycardia, dry mouth, blurred vision, and urinary retention).
Finally, the use of salicylates in children is well-recognized cause of Reyes syndrome, a condition
associated with fulminant hepatic failure. Suspected poisoning in children result in thousands of
attendances at A&E department each year. Various drugs cause specific signs when taken in overdose.
Careful examination of the child should follow A, B, C protocol.
4) Theme: Chemotherapy
Select the most appropriate chemotherapeutic agent from the above list that corresponds with
the following actions and side effects:
1) A drug disrupting microtubule leading to loss of deep tendon reflexes. J. Vincristine
2) A drug inhibiting initiation of DNA synthesis resulting in conjunctivitis
and cerebellar toxicity.

G. Cytosine arabinoside

3) A drug resulting in dose related lung damage.

D. Bleomycin

Comments:
All cytotoxic agents have adverse effects. General side-effects include nausea,
vomiting and bone marrow suppression, alopecia and stomatitis. More specific sideeffects are listed in the questions above. Alkylating agents such as Cyclophosphamide
may cause a haemorrhagic cystitis. Asparaginase results in a dose related Pancreatitis.
The anthracyclines e.g. Doxorubicin and Daunorubicin are cardiotoxic, which is often
very difficult to detect. Monitoring with echocardiograms is advised. Vincristine is an
alkaloid agent and results in sensory motor neuropathy with long-term use. It also has
an affect on the autonomic system resulting in severe constipation and paralytic ileus.
It may also result in sensory changes with parathesis progressing to loss of tendon
reflexes. Bleomycin is the main drug resulting in lung damage and occurs in up to 10%
of patients. The damage is dose related.

Pediatrics

5) Theme: Childhood malignancies


A. Burkitt's nasopharyngeal carcinoma
B. Ewing sarcoma
C. Hepatoblastoma
D. Hodgkin's lymphoma
E. Melanoma

F. Nesidio-blastosis
G. Neuroblastoma
H. Teratoma
I. Wilm's tumour
J. Xeroderma pigmentosum

Select the most appropriate oncological diagnosis from the above list of options to explain the
following scenarios
1) A 13-year-old boy presents with abdominal pain. An abdominal mass
C. Hepatoblastoma
is found. This boy has a past medical history of biliary atresia.
2) A malignant transformation of a skin resulting from an autosomal
recessive condition.

J. Xeroderma pigmentosum

3) A baby on the neonatal unit is found to be hypoglycaemic.

F. Nesidio-blastosis

Comments:
Hepatoblastoma is a malignant liver tumour. It may present with anorexia, abdominal mass and jaundice.
It is associated with cirrhosis as well as biliary cirrhosis secondary to biliary atresia. Xeroderma
pigmentosa is an autosomal recessive condition of defective DNA repair. Skin damage arise and
malignant transformation results at multiple sites. Nesidio-blastosis is also known as beta cell endocrine
benign hyperplasia and affects the pancreas. Diagnosis is made by hypoglycaemia with high serum
Insulin and treatment is by Pancreatectomy.

6) Theme: SYNDROMES
A. Down's syndrome
B. Turner's syndrome
C. Noonan's syndrome
D. Klinefelter's syndrome
E. Treacher-Collins syndrome

F. Marfan's syndrome
G. Sotos syndrome
H. Klippel-feil syndrome
I. Laurence Moon Biedl syndrome
J. MCune Albright syndrome

Which is the most applicable diagnosis for the following children.


1) A 3-year-old boy with delayed motor development is reviewed regularly in the child development clinic.
He attends for routine annual blood tests to check his thyroid function.
A. Down's syndrome
Note:
Children with Downs syndrome are screened yearly for the development of hypothyroidism. They should
also be regularly assessed for the development of myopia and hearing loss. These children have a
predisposition to acute lymphoblastic leukaemia (ALL), Alzheimers-like dementia and Atlanto-axial
subluxation.
2) A 16-year-old girl is being investigated for delayed menarche and short stature. In her notes it is
recorded that she had swollen feet at birth but she was lost to follow-up.
B. Turner's syndrome
Note:
A karyotype should be performed in all girls presenting in this way. Pedal lymphoedema at birth is a
known presentation. Other associated features include coarctation of the aorta, infertility, hypothyroidism
and horse shoe kidney.
3) A girl of 6 years has been found to have hypertension and coarctation of the aorta. She has been
referred to the growth clinic because of short stature.
B. Turner's syndrome
Note:
A karyotype should be performed in all girls presenting in this way. Pedal lymphoedema at birth is a
known presentation. Other associated features include coarctation of the aorta, infertility, hypothyroidism
and horse shoe kidney.

Pediatrics

7) Theme: PRECOCIOUS PUBERTY


A. Craniopharyngioma
B. Exaggerated adrenarche
C. McCune Albright syndrome
D. Hypothalamic hamartoma
E. Thelarche variant

F. Neurofibromatosis type 1
G. Granulosa-theca cell tumour
H. Hypothyroidism
I. Premature thelarche
J. Congenital adrenal hyperplasia

Which is the most likely diagnosis in the following cases:


1) A 5-year-old girl presents with rapid breast development, breast stage 4, over a few months and an
abdominal mass. There was no pubic hair or cliteromegaly.
Note: G. Granulosa-theca cell tumour
The rapid onset of puberty is indicative of a tumour. In this case the alpha-fetoprotein tumour marker will
be raisedG. Symptoms result from the very high oestradiol levels.
2) An 8 year old boy presents with testicular volumes of 8 mls bilaterally, pubic hair stage 3 and genital
stage 3. He has long-standing constipation, deteriorating school performance and increased weight.
Note: H. Hypothyroidism
Long standing hypothyroidism can cause precocious puberty through high levels of TRH driving
gonadotrophin secretion and cross reactivity of the FSH receptor with TRH.
3) A 2year old girl presents with isolated bilateral breast development. Examination is otherwise normal
as is her height velocity. Subsequent investigation demonstrates undetectable oestradiol levels and a
prepubertal response to an LHRH test.
Note: I. Premature thelarche
There may be a slight FSH predominance on the basal gonadotrophin levels. Premature thelarche is selflimiting and thought to be the result of the body being exquisitely sensitive to low levels of oestradiol.

8) Theme: Drugs During Pregnancy


A. Amino glycosides
B. Aspirin
C. Beta Blockers
D. Lithium
E. Phenytoin

F. Pethidine
G. Sodium Valproate
H. Sulphonamides
I. Thiazide diuretics
J. Warfarin

Select the drug from the above list of options that is most likely to be associated with the
following effects if taken during pregnancy:
1) This drug when taken in late pregnancy may result in the baby developing jaundice
H. Sulphonamides
2) This drug when taken in pregnancy may result in premature closure of the ductus arteriosis.
B. Aspirin
3) Prolonged use of this drug may result in a permanent hearing impairment.
A. Aminoglycosides

Comments:
Sulphonamides taken in late gestation may compete with Bilirubin for protein binding
sites resulting in a neonatal hyperbilirubinaemia.
Aspirin has been associated with premature closure of the ductus arteriosis as well as
haemorrhagic disorders.
Aminoglycosides (such as Gentamicin) infused over a prolonged period during
pregnancy will result in ototoxicity in approximately 2-3 % of cases.

Pediatrics

9) Theme: OBESITY
A. Type I diabetes
B. Pseudohypoparathyroidism
C. Persistent hyperinsulinaemic hypoglycaemia of infancy
D. Asthma treated with Fluticasone
E. Simple obesity

F. Prader Willi Syndrome


G. Type 2 diabetes
H. Hypothalamic tumour
I. Cerebral palsy
J. Hypothyroidism

For each of the following patients, choose the most likely cause for their obesity.
1) A 13-year-old boy presents with polyuria and polydipsia. There have been no previous concerns
regarding his health and development. He has mild asthma treated with prophylaxis in modest doses
over the winter months only. On examination he has signs of acanthosis nigricans.
G. Type 2 diabetes
2) A 3-month-old baby presents with a generalized convulsion. He has fed voraciously since birth,
consuming in excess of 250 mls/milk/kg/day. Initial investigations demonstrate a serum calcium
concentration of 2.4 mmol/L (normal) and a serum glucose concentration of 1.4 mmol/L.
C. Persistent hyperinsulinaemic hypoglycaemia of infancy
3) A 9-year-old boy, previously well, presents with a 6-month history of increasing behavioural
disturbances, increasingly severe headache and a convulsion.
H. Hypothalamic tumour

Comments:
1. Type 1 DM is unlikely to present with obesity but rather weight loss. Acanthosis nigricans is indicative
of insulin resistance and is found typically in the axillae and around the nape of the neck. These findings
suggest type 2 DM i.e. insulin resistance. However polydipsia and polyuria suggest insulinopenia and a
failing pancreas which may occur in the later stages of type 2 DM. 2. PHHI is a defect of the K+ ATP
channels in the beta cell of the pancreas. There is autonomous secretion of insulin that is independent of
the serum glucose concentration. These babies will have detectable insulin at the time of hypoglycaemia,
which would not occur normally. Treatment is with diazoxide and chlorothiazide, some require
pancreatectomy. 3. Behavioural disturbance, headache and seizures suggest intracranial pathology.
Coupled with obesity would suggest a hypothalamic tumour as this area of the brain is involved in
appetite regulation.

10) Theme: WEAKNESS IN THE LOWER LIMBS


A. Guillain-Barre syndrome
B. Transverse Myelitis
C. Juvenile Multiple sclerosis
D. Chronic fatigue syndrome
E. Spinal tumour

F. Dermatomyositis
G. Congenital myopathy
H. Duchenne muscular dystrophy
I. Fascioscapulohumeral dystrophy
J. Spinal abscess

For these children with lower limb weakness select the most likely diagnosis.
1) A 15-year-old boy started walking at 18 months of age. He has mild ptosis, absence of facial
expression and neck weakness. His mother has similar symptoms
I. Fascio-scapulo-humeral dystrophy
Note:
This is a description of fascio-scapular-humeral dystrophy, which typically presents in the teenage years.
The family history does not suggest an X-linked disorder rather autosomal dominant.
2) A 6-year-old child presents with acute onset symmetrical flaccid paralysis initially affecting the legs
then involving the trunk, and now some mild weakness in the arms. 2 weeks prior to this he had been
unwell with URTI and fever. On examination there is minimal movement with gravity eliminated in all
muscle groups in the lower limbs, deep tendon reflexes are absent, plantar responses are up-going.
Distal sensory losses are demonstrable. The CSF shows raised protein but no pleocytosis.
A. Guillain-Barre syndrome
Note:

Pediatrics

The description is of Guillain-Barre syndrome, which may be preceded by an URTI. There are lower
motor neurone signs but also with absent reflexes due to progressive distal demyelination. Sometimes
there is cranial nerve involvement (Miller Fisher syndrome)
3) A 9-year-old boy has a gradual onset of proximal weakness in the pelvic girdle and to a lesser extent
in the shoulder girdle also. There is a faint violaceous rash over the eyelids and some telangiecasia over
the finger knuckles. He has a low-grade fever and the thigh and buttock muscles are notably tender on
examination.
F. Dermatomyositis
Note:
Proximal myopathy, heliotrope rash over eyelids and the description of Gotrens papules over the
knuckles indicate dermatomyositis.

11) Theme: DELAYED MILESTONES


A. 4 weeks
B. 2 months
C. 7 months
D. 10 months
E. 12 months

F. 18 months
G. 2 years
H. 3 years
I. 5 years
J. 6 years

At what age would you refer a child for investigation if they were unable to do the following:1) Sit unsupported
D. 10 months
2) Walk independently
F. 18 months
3) Smile
B. 2 months

Comments:
It is important to be familiar with the developmental milestones, particularly up to the age of 3 years.
Milestones are grouped under 4 headings:

Gross motor
Fine motor
Hearing and language
Social

You should know the age range for each milestone. Developmental milestones are in Illingworth: 'The
Normal Child'.
12) Theme: Renal disease
A. Acute nephritis
B. Nephrotic syndrome
C. Diabetes insipidus
D. Urinary tract infection
E. Chronic renal failure

F. Diabetes mellitus
G. Salt poisoning
H. Haemolytic uraemic syndrome
I. Barrter syndrome
J. Cystinosis

Which diagnosis is most appropriate for the following patients?


1) A 2-year-old girl has become puffy in her face and has developed swollen feet. Her urine dipstick
shows proteinuria.
B. Nephrotic syndrome
Note:

Pediatrics

Hypoalbuminaemia (serum albumin falls below 25g/L); oedema, proteinuria (exceed 2g/24 hr. or more
than 40 mg/m2/hr.)(In urine dipstick +3 +4 for proteinuria) and hypercholestrolaemia (LDL G,
triglyceridesG) are features of nephrotic syndrome. So are peritonitis (pneumococcal peritonitis due to low
immunoglobulins), renal vein thrombosis and hypotension (due to hypovolaemia & hypercoagulable state
with low antthrombin III) .
2) A 15 years old boy has felt tired, lost his appetite and has not grown in the last year. His serum urea is
20mmol/l and serum creatinine is 575Omol/l. His Hb is 9.6g/dl.
E. Chronic renal failure
Note:
Chronic renal failure may have an insidious onset and may present with slowed poor growth and delayed
puberty.
3) A 2 week old boy presents to casualty with weight loss and inconsolable crying. His serum sodium is
150mmol/l and his urine osmolality is 100 mosm/kg (normal > 600).
C. Diabetes insipidus
Note:
Hypernatraemia with concomitant low urine osmolality suggest DI. Remember it is important to establish
whether nephrogenic in origin (treatment indomethacin or chlorthiazide thiazides carbamzepine chloramphenicol) or cranial (treatment DDAVP = exogenous ADH = Desmopressin = ADH analogue).

13) Theme : Neonatal Jaundice


A. ABO incompatibility
B. Breast milk jaundice
C. Cephalhaematoma
D. Congenital biliary atresia
E. Congenital cytomegalovirus infection
F. Congenital haemolytic anaemia
G. Congenital spherocytosis

H. Galactosaemia
I. Glucose-6-phosphate dehydrogenase deficiency
J. Haemolytic disease of the newborn
K. Hypothyroidism
L. Preterm birth
M. Sepsis

A 6 day old infant is admitted with jaundice (serum bilirubin 300 micromol/l) and feeding poorly
for the past 48 hours. The infant was born at a gestational age of 36 weeks and weighed 2680g at
birth. The following are some of the clinical findings and results of investigations. In each case,
select the most likely cause of the jaundice from the list of options.
1) The infant was born after a difficult forceps delivery and the head appeared to be swollen in the right
parieto-occipital area. Jaundice developed after 24 hours. The haemoglobin is 12 g/dl; there is indirect
hyperbilirubinaemia.
C. Cephalhaematoma
o

2) The baby has a temperature of 38 C and tends to bring up its feeds. The circulating white cell count
is 18x109 /l. Microscopy of urine from a suprapubic aspirate shows large numbers of neutrophil
polymorphs.
M. Sepsis
3) The liver and spleen are palpable and there is a family history of lethargy, hepatosplenomegaly and
poor feeding. The raised bilirubin is initially unconjugated; in the second sample, the conjugated fraction
tended to rise. Urine examination is positive for reducing substances using Clinitest.
H. Galactosaemia
4) On initial assessment, petechiae and hepatosplenomegaly are noted, and on investigation,
thrombocytopenia and partially conjugated hyperbilirubinaemia are found. Chorioretinitis is noted later
and intra-cranial calcification is reported on a CT brain scan.
E. Congenital cytomegalovirus infection
5) The baby was mostly breast fed and 20% of the bilirubin is conjugated. Formula milk feeds caused a
rapid fall in bilirubin levels.
B. Breast milk jaundice

Pediatrics

Comments:
1. A cephalhaematoma is a subperiosteal swelling which can arise as a result of forceps delivery.
Complications result from anaemia due to bleeding into the haematoma, and jaundice due to haemolysis
within it
2. Sepsis likely due to urinary tract infection.
3. Galactosaemia is an autosomal recessive condition due to the absence of galactose-1-phosphate
uridyl transferase. This results in the intracellular accumulation of galactose-1-phosphate which is toxic.
Upon commencement of milk feeds the child develops jaundice, vomiting, diarrhoea and failure to thrive.
4. Congenital Cytomegalovirus infection is typically characterized by the signs described. CNS damage
from congenital CMV infection exceeds that due to any other infectious cause.
5. Breast milk jaundice is defined as : neonatal jaundice persisting for more than 28 days, with a total
bilirubin level greater than 5.9mg/dl, in an otherwise healthy, thriving, breast-fed infant.
14) Theme : Connective tissue disease
A. Ankylosing spondylitis
B. Dermatomyositis
C. Ehlers Danlos syndrome
D. Henoch Schonlein purpura
E. Juvenile chronic arthritis

F. Kawasaki disease
G. Osteogenesis imperfecta
H. Rheumatic fever
I. Scleroderma
J. Systemic lupus erythematosus

Select one of the above diagnoses that would explain the following presentations:
1) A 10 year old boy presents with arthralgia, an urticarial rash on the lower limbs and buttocks and
abdominal pain.
D. Henoch Schonlein purpura
Note:
Henoch Schonlein purpura also known as anaphylactoid purpura result in non-thrombocytopenic =
(normal plat. Count) purpura nodules, arthritis, abdominal pain and nephritis.
2) A six year old girl presents with fever, arthritis and a photosensitive rash. Dipstick of her urine reveals
proteinuria.
J. Systemic lupus erythematosus
Note:
describes a child with SLE. A multi system disorder which results in fever, malaise, arthritis, rashes
typically photosensitive rash and a butterfly rash. Kidneys are also affected resulting in
glomerulonephritis. Diagnosis is made by elevated S anti-nuclear antibody, Double stranded DNA with
muscle antibodies.
3) A 4 year old boy is admitted to casualty with fracture of her ulna after minor injury. She is noted to
have blue sclerae and hearing impairment.
G. Osteogenesis imperfecta
Note:
Osteogenesis imperfecta, a syndrome of fragile bones due to defect in the T-chain of type 1collagen,
which can be divided in to 4 separate types. There is widespread osteopaenia resulting in bone fragility,
bow legs. Individuals characteristically have blue sclerae and hearing impairment (conductive deafness).
The condition has variable inheritance with autosomal dominant and recessive forms, the latter being
more severe.
Blue sclera differential diagnosis
1) Marfan syndrome
2) Ehler-Danlos syndrome
3) Osteogenesis imperficta
4) Pseudo-xanthoma elasticum

15) Theme : NEPHROLOGY


A. Acute nephritis
B. Nephrotic syndrome
C. Diabetes insipidus
D. Urinary tract infection
E. Chronic renal failure

F. Diabetes mellitus
G. Salt poisoning
H. Haemolytic uraemic syndrome
I. Barrter syndrome
J. Cystinosis

Which diagnosis is most appropriate for the following patients?

Pediatrics

1) A 4 1/2 year old girl was admitted to hospital with a 10 day history of progressive shortness of breath
and puffiness of the face. Clinical examination revealed a raised JVP, pulse rate of 160/min, normal heart
sounds, hepatomegaly 2cm and bilateral basal lung crepitations. She was seen by her family doctor for a
sore throat 2 weeks previously.
A. Acute nephritis
Note:
The picture is one of fluid overload and with a preceding URTI making post-streptococcal
glomerulonephritis the most likely diagnosis. There is often macroscopic haematuria and hypertension.
2) A 4 month baby boy is admitted because of inconsolable crying. He had been thriving otherwise and
was well previously. He seems settled during assessment, feeds well and is afebrile. Urinalysis reveals
white cells 45/HPF, red cells 10/hpf, organisms >100,000/ml.
D. Urinary tract infection
Note:
UTI should be ruled out in fractious pyrexial infants as the symptoms are non-specific. It is important to
document whether there has been previous recent antibiotic usage and where possible a clean catch
urine sample should be taken prior to starting antibiotic in this age group. Pyuria alone is not sufficient to
make the diagnosis of UTI.
3) A 9 year old girl has chronic renal failure of unknown aetiology. She has blond hair and blue eyes and
has developed hypothyroidism. She also has cataracts that are worsening over the last 2 years.
J. Cystinosis
Note:
This is a description of cystinosis which is confirmed by having high white cell cystine levels. Deposits of
cystine may occur in the thyroid and lense. Children typically have UVWX blond hair and blue eyes.

16) Theme : Personal and Social Development


A. 3 months
B. 8 months
C. 12 months
D. 18 months
E. 2 years

F. 3 years
G. 4 years
H. 5 years
I. 6 years
J. 7 years

For each description of a child's personal and social development choose, from the list of options,
the highest development age the child is most likely to have achieved.
1) The baby is reliably reacting to sounds and is able to comply with a formal hearing test.
B. 8 month

2) A child who is starting to seek adequate role models and assumes feelings and habit of thought.
G. 4 years
3) Is friendly to strangers but is not yet ready to play games such as peek-a-boo.
B. 8 months

Comments:
This question relates to the personal and social aspects of normal development.
1) Between 8- 10 months babies become anxious when separated from the main carer, usually the mother
for example if mother leaves the room.
2) Between 9-10 months the infant is able to wave bye bye and enjoys games such as peek-a-boo.
3) Between 18-24 months most children start to express their toilet needs.
4) Up until the age of 3 play is usually solitary, playing alongside their peers.
5) By 4 years the children usually start to role play for example pretending to be the same sex parent and
play involves other children in parallel first and then interactively at around this age.

Pediatrics

17) Theme : Emergency treatment


A. 100% Oxygen plus nebulised Salbutamol
B. 100% oxygen, adrenaline and hydrocortisone
C. Adrenaline IV
D. Dexamethasone
E. Hyperbaric oxygen

F. Intubation
G. IV Hydrocortisone plus oral anti-Histamines
H. Paralysis with Sodium Thiopentone
I. Skin desensitisation
J. Supportive treatment

Select the most appropriate emergency treatment for the following cases
1) A 13 year old boy is brought to A&E with a grossly swollen face and difficulty breathing following a
bee sting. YZ[\ Y]^_
B. 100% oxygen, adrenaline and hydrocortisone
2) A 2 year old boy presents with a barking cough and stridor.
D. Dexamethasone
3) A 4 month old baby with cough fever and mild respiratory distress has widespread crackles on
auscultation.
J. Supportive treatment

Comments:
1) describes an anaphylactic reaction also known as angioneurotic oedema. In this case there is difficulty
breathing i.e. life-threatening. Treatment includes 100% oxygen, IM adrenaline and IV Hydrocortisone.
Intubation may be necessary.
2) The second case has croup as characterized by the barking cough. Humidified oxygen can help for
mild cases but more severe cases require dexamethasone.
3) The principles of management for bronchiolitis includes supportive treatment only. There is no specific
treatment other than humidified oxygen; broncho dilator therapy may be of some use.
18) Theme : Fine Motor Development
A. 3 months
B. 6 months
C. 9 months
D. 12 months
E. 18 months

F. 2 years
G. 3 years
H. 4 years
I. 5 years
J. 6 years

For each description of a child's fine motor development choose, from the list of options, the
highest development age the child is most likely to have achieved.
1) Can copy a circle and a cross and climbs and descends stairs, one foot per step.
H. 4 years
2) A child can scribble and draw a straight line but not yet a circle.
F. 2 years
3) Spontaneously scribbles Y`abW with a crayon UcWdae and is starting to build a tower.
E. 18 months

Comments:
This question relates to the fine motor area of development.
1) At 4 months the infant is able to grasp big objects and move them in to the mid-line.
2) At 6 months the baby can grasp an object such a rattle and starts to transfer it from hand to
hand. He or she also starts to discover the rest of his or her body parts.
3) The pincer grip develops between 6 or 9 months and by 10 months the fine pincer grip is
achieved with the baby being able to pick up small pellet like objects between thumb and
forefinger.
4) At 18 months the baby starts to spontaneously scribble if offered a crayon for example
5) At 24 months the child is able to copy a straight line.
6) By age 3 years the child can copy a circle.

Pediatrics 10

7) By 4 years can copy cross.


8) By 4 years can copy a square.
9) By 5 years of age can copy a triangle
10) A diamond at about 6 years of age.
11) At the age of 3 the child starts to draw people usually starting as a head and stick
extremity person and as development progresses the person becomes more sophisticated.

19) Theme : Causes of vomiting


A. Cyclical vomiting
B. Duodenal atresia
C. Over feeding
D. Meckel's diverticulum
E. Ulcerative colitis

F. Meconium ileus
G. Gastro-enteritis
H. Gastro-oesophageal reflux
I. Pyloric stenosis
J. Urinary tract infection

Select one option from the list above that is most suitable for the following patients
1) A 2 day old breast fed baby is vomiting after each feed. Abdominal x-ray demonstrates a double
bubble.
B. Duodenal atresia
Note:
Duodenal atresia is the most common cause of duodenal obstruction in infancy. It is frequently found in
babies with Down's syndrome. Babies present with bilious or non-bilious vomiting and a double bubble
are seen on abdominal x-ray. This is the result of distension of the stomach and duodenum with a
constricting pylorus between them.

2) A one year old boy with a history of intermittent bleeding pr presents with features of an interception
D. Meckel's diverticulum
Note:
Meckel's diverticulum is most common under the age of 2. The Meckel's diverticulum is remnant of the
vitilo-intestinal duct and can present with features of intussusception or diverticulitis. Commonly presents
with painless bleeding and is diagnosed by a Meckel's scan.

3) A 4 month old baby who is thriving has persistent vomiting which is occasionally blood stained.
H. Gastro-oesophageal reflux
Note:
Gastro-oesophageal reflux. Reflux of gastric contents in to the oesophagus is a common event in infancy.
Once the abdominal segment of the oesophagus lengthens in the first few months of life reflux improves.

20) Theme : Drugs During Pregnancy


A. Aminoglycosides
B. Aspirin
C. Beta Blockers
D. Lithium
E. Phenytoin

F. Pethidine
G. Sodium Valproate
H. Sulphonamides
I. Thiazide diuretics
J. Warfarin

Select the drug from the above list of options that is most likely to be associated with the
following effects if taken during pregnancy:
1) This drug taken in early pregnancy may result in Ebstein's anomaly in 3% of cases.
D. Lithium
2) This drug may result in a neural tube defect in approximately 2% of cases.
G. Sodium Valproate
3) This drug, when taken in pregnancy may result in a baby being born with cranio-facial abnormalities,

Pediatrics 11

growth impairment and learning difficulties.


E. Phenytoin

Comments:
1) Lithium, a drug used in the treatment of bipolar affective disorder. If given in early
pregnancy is associated with a wide variety of cardiac defects with approximately 8%
having severe cardiac disease, Ebstein's anomaly being the most common (3% of all
cases).
2) Sodium Valproate tends to induce a neural tube defect (spina bifida) in
approximately 2% of pregnancies and therefore should be avoided. It is also associated
with abnormalities of the orofacial and digits, compromising the foetal Valproate
syndrome.
3) Phenytoin is well known for causing the foetal hydantoin syndrome, compromising
cranio-facial abnormalities and learning difficulties as well growth impairment.
21) Theme : Abdominal Pain
A. Acute gastro-enteritis
B. Appendicitis
C. Constipation
D. Diabetes Mellitus
E. Henoch Schonlein Purpura

F. Mesenteric Adenitis
G. Nephroblastoma
H. Pancreatitis
I. Sickle Cell crisis
J. Wilm's Tumour

Select one option from the list above that is most suitable for the following patients
1) A 12 year old boy with a 3 week history of coryza is brought to A&E complaining of severe abdominal
pain. He has swollen ankles and nodules on his buttocks, which do not disappear with pressure.
E. Henoch Schonlein Purpura
Note:
Henoch Schonlein purpura also known as anaphylactoid purpura has an unknown aetiology. Children
present with abdominal pain, possibly melaena due to haemorrhage and oedema of the gut wall. They
may have a flitting arthritis affecting large joints. The nephritis presents with microscopic haematuria and
proteinuria and the macular, papular, purple rash is rather characteristic with buttocks and extensive
surfaces of the legs and arms being affected.

2) A 10 year old girl has just returned from Thailand. She is brought to history of severe diarrhoea
associated with abdominal pain. Her mother, a known diabetic, has similar symptoms.
A. Acute gastro-enteritis
Note:
relates to a child with gastroenteritis. In the developed world gastroenteritis is very common and usually
mild. 60% of cases are due to viruses for example Rotavirus. In bacterial gastroenteritis; fever is common
and children present with colicky abdominal pain, vomiting, diarrhoea and possibly dehydration.
3) A 12 year old girl with a 2 month history of weight loss and increased appetite weight loss is brought to
Casualty with severe abdominal pain. She denies any history of vomiting or excessive exercise. Her
mother says that the child has been wetting her bed over the past 4 months.
D. Diabetes Mellitus
Note:
describes a child with diabetes mellitus. The commonest metabolic / endocrine problem in childhood.
Diabetes mellitus results from low Insulin levels resulting in abnormal metabolism of carbohydrate,
protein and fats. Children present with polyuria, polydipsia and polyphagia. Children sometimes complain
of abdominal discomfort and there is a history of weight loss.

Pediatrics 12

22) Theme : Causes of Headaches


A. Benign intra-cranial hypertension
B. Cerebral tumour
C. Cluster headache
D. Encephalitis
E. Extradural haematoma

F. Meningitis
G. Migraine
H. Subdural haemorrhage
I. Sub arachnoid haemorrhage
J. Tension headache

Select the most appropriate diagnosis from the above list that would explain the following cases.
1) A 16 year old girl develops headache and vomiting. She has a fever and develops a non-blanching
rash.
F. Meningitis
2) A 17 year old male with a 4 day history of cough and fever presents with altered sensorium and
vomiting.
D. Encephalitis
3) A 10 year old girl with a 6 month history of episodic abdominal pain presents with a severe headache,
a complete ophthalmoplegia which lasts several days.
G. Migraine

Comments:
1) Bacillus Meningitis is a severe infection that carried serious morbidity and mortality. It may present
suddenly or insidiously. Most commonly symptoms start off as being non-specific, for example fever,
anorexia, irritability and later develop in to a rash and shock. More specific signs include neck stiffness
and seizures.
2) Encephalitis refers to inflammation of the brain and children present with an encephalopathy. Initial
symptoms may be non-specific or flu-like followed by changes in mental state, seizures and
complications include cerebral oedema, cardiovascular instability, fluid and electrolyte disturbances.
Herpes viruses have been implicated as well as other viruses and often there is a devastating
consequence.
3) Migraine is a recurrent headache with symptom free interval. Other symptoms include abdominal pain,
nausea or vomiting, aura, visual sensory or motor (photophobia may also occur). Other visual
disturbances may take the form of blurred vision, fortification spectra and flashing lights. Complicated
migraine occurs when there are neurological signs during a headache that may persist after the
headache has terminated. For example hemiplegic migraine.

23) Theme : Dyspnoea in children


A. Acute severe asthma
B. Cardiac failure
C. Costochondritis
D. Diabetic ketoacidosis
E. Hysterical over-breathing

F. Inhalation of foreign body


G. Pleura effusion
H. Pulmonary embolism
I. Tension pneumothorax
J. Pleural effusion

Select the most appropriate diagnosis from the options above that best explains the following
cases:
1) A 3 month old baby presents with tachycardia and tachypnoea with widespread crepitations
B. Cardiac failure
2) A 3 year old presents with cough and haemoptysis. Examination reveals reduced breath sounds over
one lung and a localized wheeze.
F. Inhalation of foreign body
3) A boy presents with a dry cough, lethargy and shortness of breath. Examination reveals reduced
breath sounds over one side which is dull to percussion.
J. Pleural effusion

Pediatrics 13

Comments:
Heart failure A number of congenital heart lesions will result in cardiac failure, commonly in the first
year of life. The baby becomes breathless particularly after crying or feeding. The baby may be having
difficulty completing feeds and as a result there may be a problem with poor weight gain. Babies also are
seen to have sweating as a prominent symptom. On examination the baby may be tachycardic and
tachypnoenic. A murmur may be present or a gallop rhythm. Oedema may be seen around the
dependant parts of the body. Pulmonary crepitations may also be heard. It may be possible to pick up
heart enlargement clinically and chest x-ray may reveal cardiomegaly and plethoric lung fields. In the
second case the unilateral nature of the reduced breath sounds and wheeze strongly suggest
inspissation of a foreign body. The final case has again unilateral signs but on this occasion the dullness
to percussion suggest a pleural effusion.
24) Theme : Abdominal diseases of Childhood
A. Aerophagia
B. Alpha 1 antitrypsin deficiency
C. Galactosaemia
D. Hirschsprung's disease
E. Intussusception

F. Lactose intolerance
G. Mesenteric adenitis
H. Reyes syndrome
I. Thalassaemia
J. Wilson's disease

Select one of the above options that is the most appropriate diagnosis for the following cases:
1) Usually presents with chronic liver disease often in association with neurological features.
J. Wilson's disease
Note:
describes a child with Wilson's disease also known as hepatolenticular degeneration. It is an autosomal
recessive condition characterized by cirrhosis of the liver, degenerative changes of the brain and KayserFleischer rings on the cornea. It is as a result of low Caeruloplasmin and low Copper excretion.
2) A 9 month old baby presents with a sudden onset of abdominal pain and loose stools which have the
appearance of red currant jelly.
E. Intussusception
Note:
Intussusception is an invagination of the bowel in to an adjacent segment. It presents as paroxysmal pain
occasionally with vomiting and palpation may reveal a sausage shaped mass : in the right upper
abdomen. A delay in diagnosis results in the child passing blood stained mucus rectally some times
known as red currant jelly. Intussusception can sometimes be reduced hydro-statically, if unsuccessful
surgery is required.
3) A 4 day old infant presents with severe abdominal distension and feculent vomiting
D. Hirschsprung's disease
Note:
Hirschsprung's disease is absence of ganglia cells in part or all of the wall of the colon. The child
presents with bilious or feculent vomiting, constipation and distension.

25) Theme : Gross motor development


A. 1 month
B. 2 months
C. 4 months
D. 6 months
E. 8 months

F. 10 months
G. 1 year
H. 18 months
I. 2 years
J. 3 years

For each description of a child's gross motor development choose, from the list of options, the
highest development age the child is most likely to have achieved.
1) A baby can crawl, weight bare and is starting to cruise.
F. 10 months
2) A baby rolls from front to back and back to front however is not yet sitting unsupported.
D. 6 months

Pediatrics 14

3) A child ascends stairs using alternating feet and is able to stand on one foot for a few seconds.
J. 3 years

Comments:
This question relates to gross motor skills area of development.
1. At 3 months when placed prone an infant can lift his head with his arms extended.
2. By 4 months head lag is lost and this is evident when raised from supine to the sitting position.
3. At 5 months the baby starts to roll over and
4. At 6 months the baby is starting to sit up with support for example cushions.
5. Sitting unsupported is usually seen by 6-7 months
6. At 9 months the baby can start to crawl.
7. Cruising is usually achieved by about 10 months.
8. At 12 months the baby can raise ijVk independently and often starts to take a few steps alone.
9. At 18 months the toddler can run although this is usually in a tiff fashion. He or she can ascend
stairs with assistance
10. At 20 months the child can climb the stairs holding on to a rail.
11. At 3 years the child can ascend stairs in an adult fashion using alternating feet per step however
is unable to descend the stairs in such a fashion until 4 years of age.

26) Theme : Treatment of Diarrhoeal illness


A. Metronidazole
B. Gluten free diet
C. No action
D. Steroids
E. Lactose free diet

F. Pancreatic enzyme supplements


G. Vitamin D
H. High fibre diet
I. Breast milk allergy
J. Praziquantel

Select one option from the list above that is most suitable for the following patients
1) A 2 year old girl is taken to the A&E department by her mother who says that her child has diarrhoea
and describes the stool as looking like peas and carrots.
C. No action
Note:
Describes a child with toddler diarrhoea due to a fast gut transit time. Children between the ages of 6
months and 3 years may have this complaint which manifests as a chronic diarrhoea. Children are
otherwise healthy and thriving with all investigations carried out being normal.
2) A 7 year old child presents with a 3 month history of abdominal pain. She is found to have anaemia
and colonoscopy reveals a cobble stone appearance.
D. Steroids
Note:
Relates to inflammatory bowel disease, Crohn's disease and ulcerative colitis respectively. In Crohn's
disease the colonoscopy will reveal :cobbled stone mucosal appearances with fistulae where as in
ulcerative colitis : (crypt abscesses + Collar button ulcers)are present.

3) A 12 year old girl whose height and weight are on the 3rd centile complains of diarrhoea with a history
that the stools are difficult to flush away. She has an itchy rash over her limbs.
B. Gluten free diet
Note:
relates to Coeliac disease. A sensitivity and intolerance to the gliadin protein : in rice, wheat and barley.
Children present with failure to thrive, diarrhoea, abdominal pain, anorexia and abdominal distension.
Diagnosis is suspected by elevated (anti-gliadin / anti-reticulin / anti-endomysial) antibodies and
confirmed by jejunal biopsies.

Pediatrics 15

27) Theme : TREATMENT OF CHILDHOOD CONDITIONS


A. Acyclovir
B. Cefotaxime
C. Erythromycin
D. Gentamicin
E. Immunoglobulin

F. No medication
G. Paracetomol
H. Penicillin
I. Prednisolone
J. Quinine

All of the following children present with a febrile illness. Based on the symptoms presented
below, select the single treatment option most likely to alter the clinical course of the disease in
each case. Each option may be used once, more than once, or not at all
1) A playful and active 5 year old boy presents with a low grade fever and bruising. The rest of the
examination is normal. His platelet count is 14 x 109 /L
F. No medication
Note:
Childhood thrombocytopenia may occur in the setting of viral illness.
2) A 10 year old girl comes back from school camp with a temperature, breathlessness and cough, which
has been going on for 10 days
C. Erythromycin
Note:
Outbreaks of whooping cough may in school camps. The most vulnerable children are not immunized.
3) A 5 month old baby presents with a fever, lethargy, poor feeding and a generalized convulsion. An
examination of the cerebrospinal fluid shows an elevated number of neutrophil polymorphs
B. Cefotaxime
Note:
Meningitis should be suspected in any ill baby and may present with unusual crying, poor feeding, fever,
coma, fits and vomiting. Bulging or tense anterior fontanelle, stiff neck, opisthotonus and shock may be
present. For children more than three months old cefotaxime is the treatment of choice.
4) A 3 year old Asian girl presents with a high temperature for six days, cervical lymphadenopathy,
conjunctivitis and a red tongue. There is a generalized rash, with desquamation of the skin of the fingers
and toes.
E. Immunoglobulin
Note:
Kawasakis disease is characterized by bilateral conjunctival injection, oral mucosal changes with red
sore tongue, polymorphous rash, lymphadenopathy and desquamation of palms and soles. Inflammatory
markers are elevated and treatment is with intravenous immunoglobulin.
5) A 4 year old girl presents with a temperature of 39oC having just had a generalized convulsion lasting
3 minutes; twenty minutes later, she is completely alert again. Her older brother had similar fits at the age
of 3 years
G. Paracetomol
Note:
Febrile convulsions are generalized tonic/clonic symmetrical seizures which occur with rapid rises in
temperature in children between the ages 3 months and 5 years who have no history of epilepsy or CNS
disease.

28) Theme : Fine Motor Development


A. 3 months
B. 6 months
C. 9 months
D. 12 months
E. 18 months

F. 2 years
G. 3 years
H. 4 years
I. 5 years
J. 6 years

For each description of a child's fine motor development choose, from the list of options, the
highest development age the child is most likely to have achieved.

Pediatrics 16

1) A child can copy a square and a triangle but not yet a diamond.
I. 5 years
2) A baby transfers objects to his mouth and has a fine pincer grip using thumb and forefinger.
D. 12 months
3) Can only copy a circle and builds a tower of 9 cubes.
G. 3 years

Comments:
This question relates to the fine motor area of development.
1) At 4 months the infant is able to grasp big objects and move them in to the mid-line.
2) At 6 months the baby can grasp an object such a rattle and starts to transfer it from hand to hand. He
or she also starts to discover the rest of his or her body parts.
3) The pincer grip develops between 6 or 9 months and by 10 months the fine pincer grip is achieved
with the baby being able to pick up small pellet like objects between thumb and forefinger.
4) At 18 months the baby starts to spontaneously scribble if offered a crayon for example.
5) At 24 months the child is able to copy a straight line.
6) By age 3 years the child can copy a circle.
7) By age 4 years the child can copy a cross.
8) By 4 years the child can copy a square
9) By 5 years of age a triangle,
10) A diamond at about 6 years of age.
At the age of 3 the child starts to draw people usually starting as a head and stick extremity person and
as development progresses the person becomes more sophisticated.

29) Theme : Poisoning


A. Antipsychotics
B. Benzodiazepines
C. Ethylene glycol
D. Methanol
E. Opiates

F. Organophosphates
G. Salicylates
H. Sympathomimetic
I. Tricyclic antidepressants
J. Volatile solvents

Select the most appropriate agent from the above list that would elicit the following toxic effects:
1) Blindness, pulmonary oedema, metabolic acidosis, shock.
D. Methanol
2) Coma, hypotension, hypoventilation, reduced muscle tone and diminished reflexes.
B. Benzodiazepines
3) Dysphagia, Dysphoria, Trismus, oculogyric crisis
A. Antipsychotics

Comments:
The relevant feature of the first case is the blindness and acidosis which is probably due to Methanol,
itself a weak acid but it causes lactic acidosis and optic neuritis. The second case has features of CNS
depression and Benzodiazepines are typical. This should respond rapidly to Flumazenil. The third
scenario with oculogyric crisis suggests antipsychotics.

30) Theme : chest pain in children


A. Acute severe asthma
B. Cardiac failure
C. Costochondritis
D. Diabetic ketoacidosis
E. Hysterical over-breathing

F. Inhalation of foreign body


G. Pleura effusion
H. Pulmonary embolism
I. Tension pneumothorax
J. Unilateral pneumonia

Pediatrics 17

Select the most appropriate diagnosis from the options above that best explains the following
cases:
1) 14 year old presents to Accident & Emergency with a sudden onset of chest pain and shortness of
breath. Examination reveals reduced expansion and hyperresonance over one side with reduced breath
sounds.
I. Tension pneumothorax
2) A 17 year old presents with a sudden onset of chest pain. Examination reveals a sinus tachycardia
and a pleural rub.
H. Pulmonary embolism
3) A 13 year old boy presents with tachypnoea and tachycardia. Examination of the chest reveals poor
air entry, however no crackles or wheeze. He has a pulsus paradoxus.
A. Acute severe asthma

Comments:
Tension pneumothorax may result following trauma. It is a potentially life threatening event with air
accumulating under the pleura in the pleural space, which pushes the mediastinum across the chest.
Children may be shocked and will be hypoxic with reduced air entry and hyperresonance. Distended
neck veins may also be visible.
Pulmonary embolism is uncommon in infants and children. It may arise in children as a result of surgery
for example scoliosis surgery. It may also occur following prolonged inactivity. In adolescents drug abuse,
recent abortion or the oral contraceptive may be predisposing problems. Clinical manifestations may
suggest pneumonia. Dyspnoea is common associated with pain. There may be no physical signs
although the embolism is sufficiently large there may be a pleural friction rub.
The clue in the third case the relatively silent chest associated with pulsus paradoxus. This would
suggest severe asthma. Other
Causes of pulsus paradoxus include : (pericardial tamponade /effusion/ constriction and myocarditis).

31) Theme : Abdominal Disease


A. Acute gastro-enteritis
B. Appendicitis
C. Constipation
D. Diabetes Mellitus
E. Henoch Schonlein Purpura

F. Mesenteric Adenitis
G. Nephroblastoma
H. Pancreatitis
I. Sickle Cell crisis
J. Wilm's Tumour

Select one option from the list above that is most suitable for the following patients
1) A 9 year old Italian girl has severe abdominal pain. She is found to have sclerae and painful fingers.
I. Sickle Cell crisis
Note:
Sickle cell disease is a haemoglobinopathy caused by substitution of Valine for Glutamic acid at the 6th
position of the Beta chain. It results in a haemolytic anaemia resulting in vaso-occlusive episodes leading
to splenic infarction and dactylitis.
2) An 11 year old boy with a 12 hour history of anorexia and polyuria complains of severe abdominal
pain. On examination he has a temperature of 38C and is tender posteriorly on pr examination. BM is
4mmol per litre.
B. Appendicitis
Note:
describes acute appendicitis. The commonest acute surgical emergency of childhood. Usually seen in
children over the age of 5. Children present with a temperature, abdominal pain and guarding in the right
iliac fossa, rebound tenderness, fever and vomiting. It is as a result of mucus obstructing the lumen
causing ischaemia of the mucosa and ultimately perforation.
3) An 11 year old boy with cystic fibrosis presents with recurrent attacks of abdominal pain, vomiting and
fever.
H. Pancreatitis

Pediatrics 18

Note:
relates to description of chronic Pancreatitis in a child with cystic fibrosis. It is also a feature of chronic
hyperlipidaemia or congenital anomalies of the biliary retract or pancreas. Children present with vomiting
and abdominal pain which radiates through tot he back. Clinical suspicion is confirmed by elevated
amylase levels and calcification of the pancreas on abdominal x-ray. Treatment is supportive

32) Theme : Abdominal diseases of Childhood


A. Aerophagia
B. Alpha 1 antitrypsin deficiency
C. Galactosaemia
D. Hirschsprung's disease
E. Intussusception

F. Lactose intolerance
G. Mesenteric adenitis
H. Reyes syndrome
I. Thalassaemia
J. Wilson's disease

Select one of the above options that is the most appropriate diagnosis for the following cases:
1) A 10 year old child develops severe vomiting and becomes encephalopathic during a viral illness.
H. Reyes syndrome
Note:
Reyes syndrome is an acute disease of the liver often associated with a viral infection or Salicylate
toxicity . Laboratory findings include abnormal liver and muscle enzymes. Treatment is supportive.
2) A 3 year old child with a 2 day history of headache and pharyngitis complains of severe abdominal
pain.
G. Mesenteric adenitis
Note:
Mesenteric adenitis commonly accompanying upper respiratory tract infections and is due to
inflammation in the abdominal lymph nodes. There is often a history of pharyngitis and abdominal pain.
Examination often reveals cervical lymphadenopathy and central tenderness.
3) A 28 day old jaundice baby with early onset vomiting and diarrhoea.
C. Galactosaemia
Note:
Galactosaemia results as a result of galactose 1 phosphate uridyl transferase. It accumulates in kidneys,
liver and brain and clinical manifestations include jaundice, hepatomegaly, hypoglycaemia, vomiting,
seizures and poor weight gain. Diagnosis is made by identifying urinary galactose. Treatment is dietary
adjustment.

33) Theme : Skin lesions


A. Caf au lait spots
B. Capillary haemangioma
C. Port-wine stain
D. Shagreen patch
E. Adenoma sebaceum

F. Cutis Marmorata
G. Herald's patch
H. Peri-oral pigmentation
I. Hypopigmentation
J. Tuber

Select one option from the list above that is most suitable for the following patients
1) A child has an abundance of birth marks associated with axillary freckling.
A. Caf au lait spots
Note:
relates to Neurofibromatosis type 1. Caf au lait lesions are brown pigmented marks found in abundance
in Neurofibromatosis as well as other genetic conditions for example Tuberous sclerosis.
2) A child with learning difficulties is described as having goose like flesh.
D. Shagreen patch
Note:
relates to Tuberous sclerosis. It is a condition where by hamartomatous lesions become evident in early
childhood. Seizures and learning difficulties are common and skin manifestations include fibrous
angiomatous lesions in the naso labial folds known as adenoma sebaceum. White macules are seen
over the trunk known as Ash leaf macules and Shagreen patches resemble goose like flesh.

Pediatrics 19

Subungual fibromata are also common.


3) A child with a history of Iron deficiency anaemia presents with per rectal bleeding and is found to have
this characteristic appearance.
H. Peri-oral pigmentation
Note:
refers to Peutz-Jeghers syndrome characterized by peri-oral pigmentation. A history of Iron deficiency
anaemia, colonoscopy may reveal polyps.

34) Theme : Personal and Social Development


A. 3 months
B. 8 months
C. 12 months
D. 18 months
E. 2 years

F. 3 years
G. 4 years
H. 5 years
I. 6 years
J. 7 years

For each description of a child's personal and social development choose, from the list of
options, the highest development age the child is most likely to have achieved.
1) A baby shows stranger awareness, is able to wave and clap his hands and is starting to spoon feed
himself.
C. 12 months
2) Can dress and undress including buttons and can imitate the construction of bridges and steps with
blocks.
G. 4 years
3) A child is starting to indicate the need to sit on the potty, play is usually solitary at this age and the
child tries to control objects over other children.
E. 2 years

Comments:
This question relates to the personal and social aspects of normal development.
1) Between 8- 10 months babies become anxious when separated from the main carer, usually the
mother for example if mother leaves the room.
2) Between 9-10 months the infant is able to wave bye bye and enjoys games such as peek-a-boo.
Between 18-24 months most children start to express their toilet needs.
3) Up until the age of 3 plays is usually solitary, playing alongside their peers.
5) By 4 years the children usually start to role play for example pretending to be the same sex parent and
play involves other children in parallel first and then interactively at around this age.

35) Theme : Genital disorders


A. Aarskog syndrome
B. Fragile X
C. Gonadal dysgenesis
D. Hermaphroditism
E. Polycystic ovaries

F. Testicular feminisation syndrome


G. XO syndrome
H. XXX syndrome
I. XXY syndrome
J. XYY syndrome

Select the most appropriate diagnosis from the list above that best describes the following cases
1) A 12 year old girl has seizures, depression and a conduct disorder.
H. XXX syndrome
Note:
describes a girl with XXX syndrome. Individuals are tall and thin. They have behavioural problems and
are mildly dysmorphic with epicanthic folds and hypertelorism. Seizures are common

Pediatrics 20

2) A 16 year old female presents with obesity, oligomenorrhoea and hirsutism.


E. Polycystic ovaries
Note:
relates to Stein-Leventhal also known as polycystic ovarian syndrome. During adolescence there is
change in body habitus and there is excess hair grown and chronic and ovulation.
3) A 14 year old phenotypic female presents with primary amenorrhoea. She has normal female external
genitalia but intra-abdominal testicles with an XY karyotype.
F. Testicular feminisation syndrome
Note:
The third case has testicular feminisation, a condition associated with testosterone receptor defects that
result in insensitivity to testosterone with resultant female genitalia.

36) Theme : NEONATAL JAUNDICE


A. Physiological jaundice
B. Biliary atresia
C. Hypothyroidism
D. Rhesus incompatibility
E. Congenital spherocytosis

F. Congenital cytomegalo virus infection


G. Galactosaemia
H. Glucose-6-phosphate dehydrogenase deficiency
I. Cystic fibrosis
J. Fructose intolerance

For each of the following jaundiced babies, select the most likely cause.
1) A two-week-old full term baby with a birth weight of 1.9 kg. He has gained a small amount of weight
already. On examination he has a white reflex on shining a bright light into both pupils, moderate
jaundice, a pansystolic murmur at the left lower sternal edge, hepatosplenomegaly and a rash.
F. Congenital cytomegalo virus infection
Note:
Low birth weight, white reflex suggesting congenital cataracts, jaundice and hepatosplenomegaly are
indicative of a congenitally acquired infection. The rash is likely to be petechiae secondary to bone
marrow failure from the infection.
2) A 48 hour old baby who presents with poor feeding. She was delivered at home following a concealed
pregnancy. She was thought to be pale initially but her skin subsequently became more yellow. Her
mother had a late miscarriage one year previously and is blood group A negative. Investigations show a
haemoglobin of 10 g/dL and an unconjugated bilirubin of 426 mmol/L. The blood film shows evidence of
haemolysis but otherwise normal red cell morphology.
D. Rhesus incompatibility
Note:
The previous miscarriage has sensitized the maternal immune system to the Rhesus antigen.
Haemolysis causes an unconjugated hyperbilirubinaemia. ABO incompatibility unlikely unless the mother
is blood group O.
3) A 5 day old baby appears jaundiced but otherwise well and feeds normally. The serum bilirubin is
checked by the community midwife and found to be moderately elevated at 298 mmol/L. It is mainly
unconjugated bilirubin, and the haemoglobin is normal.
A. Physiological jaundice
Note:
This is a well baby and with only a slightly elevated unconjugated bilirubin. Levels can normally increase
to 300 mmol/L with no adverse effects on the baby. Some reports say that levels are safe up to 400
mmol/L in a term well baby.

37) Theme : Causes of vomiting


A. Gastro-oesophageal reflux
B. Intussusception
C. Meconium ileus
D. Meconium plug syndrome
E. Meckel's Diverticulum

F. Mid gut volvulus


G. Necrotising enterocolitis
H. Peptic ulcer
I. Pyloric stenosis
J. Tracheo-oesophageal fistula with atresia

Pediatrics 21

Select the most appropriate option from the above list that would account for the following
presentations:
1) A baby on the postnatal ward is seen to have a cyanotic episode during feeding.
J. Tracheo-oesophageal fistula with atresia
Note:
Tracheo-oesophageal fistula with oesophageal atresia results in excessive oral secretions. The baby
often chokes or coughs with feeds and becomes cyanotic. There is an inability to pass a nasogastric tube
and there is often a history of maternal polyhydramnios.
2) A baby born at 33 weeks develops vomiting and abdominal distension on day 3. By day 4 the
abdomen appears erythematous and the baby becomes apnoeic.
G. Necrotising enterocolitis
Note:
Necrotising enterocolitis is a life threatening condition resulting in mucosal and transmural necrosis. It is
most prevalent in low birth weight babies and the babies present with abdominal distension, gastric
retention, bloody stools and signs of a sepsis. Abdominal x-ray may reveal pneumatosis intestinalis.
3) A term baby is admitted to Special Care Baby unit with hypoglycaemic episodes. His mother is a
poorly controlled diabetic. By day 4 the hypoglycaemia has resolved but the baby has not yet opened his
bowels.
D. Meconium plug syndrome
Note:
Meconium plug syndrome is as a result of a plug of meconium causing obstruction. There is usually an
underlying disease such as cystic fibrosis, Hirschsprung's disease, infant of a diabetic mother or maternal
drug abuse.

38) Theme : Treatment of Diarrhoeal illness


A. Metronidazole
B. Gluten free diet
C. No action
D. Steroids
E. Lactose free diet

F. Pancreatic enzyme supplements


G. Vitamin D
H. High fibre diet
I. Breast milk allergy
J. Praziquantel

Select one option from the list above that is most suitable for the following patients:
1) A 7 month old baby has diarrhoea, which mother feels has been brought on by the introduction of
weaning. Mum is a known asthmatic.
E. Lactose free diet
Note:
refers to Cow's milk protein intolerance is mainly a clinical diagnosis with a history that symptoms
appeared at the time of cow's milk ingestion. The disorder is usually temporary and can be managed by
dietary adjustment.
2) A 13 year old Irish girl complains of diarrhoea. She is Iron deficient and denies any history of GI upset
after eating rice or potatoes. She has high anti-reticulin antibodies
B. Gluten free diet
Note:
refers to Coeliac disease where a gluten free diet is appropriate. High reticulin or more specifically now
anti-TTG antibodies are expected. The diagnosis is confirmed by jejunal biopsy.
3) A 10 year old boy presents with faecal urgency and bloody diarrhoea. He is also under the care of
ophthalmologists for a chronic iritis.
D. Steroids
Note:
relates to inflammatory bowel disease, Crohn's disease and ulcerative colitis respectively. In Crohn's
disease : the colonoscopy will reveal cobbled stone mucosal appearances with fistulae where as in
ulcerative colitis I crypt abscesses are present.

Pediatrics 22

39) Theme : Causes of vomiting


A. Cyclical vomiting
B. Duodenal atresia
C. Over feeding
D. Meckel's diverticulum
E. Ulcerative colitis

F. Meconium ileus
G. Gastro-enteritis
H. Gastro-oesophageal reflux
I. Pyloric stenosis
J. Urinary tract infection

Select one option from the list above that is most suitable for the following patients
1) An 8 year old girl shows signs of moderate dehydration. She has vomited all fluids of 24 hours and the
vomit is not bile stained. Her abdomen is soft and she has had 3 similar episode in the past year.
A. Cyclical vomiting
Note:
Cyclical vomiting. This is thought to be a variant of migraine. The child presents with bouts of vomiting
which may persist for days and at times a headache is associated.
2) A 12 week old infant vomits after each feed. He is developmentally normal and is fed by bottle at
260mls per kilo per day.
C. Over feeding
Note:
Over feeding frequently results in regurgitation and vomiting. This may also be accompanied by an
excessive weight gain and dietary adjustment is indicated.
3) An 11 year old boy with a 3 month history of abdominal pain and weight loss presents with bloody
diarrhoea.
E. Ulcerative colitis
Note:
The last question relates to ulcerative colitis a form of inflammatory bowel disease. In this case the child
presents with recurrent bouts of profuse bloody diarrhoea often with mucus and abdominal pain.
Associated features include arthritis, pyoderma gangrenosum, erythema nodosum and anaemia.

40) Theme : Drugs to be avoided during breastfeeding


A. Amiodarone
B. Aspirin
C. Benzodiazepine
D. Chloramphenicol
E. Heparin

F. Lactulose
G. Lithium
H. Sulfasalazine
I. Sulphonamides
J. Warfarin

Which of the above drugs when prescribed to the mother is associated with the following
abnormalities in the breast-fed infant:
1) May result in neonatal hypothyroidism.
A. Amiodarone
2) May result in drowsiness and failure to thrive.
C. Benzodiazepine
3) May result in neonatal diarrhoea.
F. Lactulose

Comments:
Administration of certain drugs to breast feeding mums may result in toxicity in the infants if the
drug enters the milk in pharmacologically significant quantities. Some drugs also inhibit the
babies sucking reflex. Other drugs inhibit lactation.
Amiodarone needs to be avoided as it may result in neonatal thyroid abnormalities as there is
a theoretical risk from the release of Iodine.
Benzodiazepine may render the infant drowsy resulting in poor growth.
Laxatives if used over a prolonged period may enter the breast milk resulting in the baby
developing diarrhoea.

Pediatrics 23

41) Theme : Paediatric endocrinology


A. 3 Beta hydroxysteroid dehydrogenase
deficiency
B. 17 Hydroxylase deficiency
C. 21 Hydroxylase deficiency
D. Adrenal hyperplasia
E. Craniopharyngioma

F. Hypoadrenalism
G. Phaeochromocytoma
H. Panhypopituitarism
I. Polyglandular endocrinopathy
J. Primary aldosteronism

Select the most appropriate diagnosis from the above options to explain the following
presentations:
1) An 8 year old boy presents with a history of enuresis. Examination reveals a bi-temporal hemianopia
and papilloedema.
E. Craniopharyngioma
Note:
refers to a child with a Craniopharyngioma which is the commonest supratentorial tumour found in
children. Children present with accelerated head growth or headaches and examination findings include
papilloedema and a bi-temporal hemianopia This tumour may result in diabetes insipidus, hypothyroidism
and growth hormone deficiency.
2) A term baby is born with ambiguous genitalia. Examination reveals a phallus with pigmented labial
scrotal folds and the baby is found to be hyponatraemic.
C. 21 Hydroxylase deficiency
Note:
describes a baby with congenital adrenal hyperplasia. 95% of cases are due to 21 Hydroxylase
deficiency and can be diagnosed by increased levels of 17OHP. The salt wasting results in
hyponatraemia (Na n) and hyperkalaemia (k G).
3) A 10 year old girl complains of headaches and vomiting. On examination she is noted to be pale and
her blood pressure measures 135/90. Numerous caf au lait lesions are found over her trunk.
G. Phaeochromocytoma
Note:
describes a child with Neurofibromatosis complicated by : phaeochromocytoma, which is a tumour
arising from the adrenal medulla or chromaffin cells. Children present with headaches, palpitations,
pallor, vomiting and night sweats. They are found to be hypertensive.

42) Theme : Drugs to be avoided during breastfeeding


A. Amiodarone
B. Aspirin
C. Benzodiazepine
D. Chloramphenicol
E. Heparin

F. Lactulose
G. Lithium
H. Sulfasalazine
I. Sulphonamides
J. Warfarin

Which of the above drugs, when prescribed to the mother, is associated with the following
abnormalities in the breast-fed infant:
1) May result in hypotonia and cyanosis.
G. Lithium
2) May result in prolonged jaundice.
I. Sulphonamides
3) May result in platelet dysfunction.
B. Aspirin

Pediatrics 24

Comments:
Administration of certain drugs to breast-feeding mums may result in toxicity in the infants if
the drug enters the milk in pharmacologically significant quantities. Some drugs also inhibit the
babies sucking reflex. Other drugs inhibit lactation.
Lithium is also to be avoided as it may result in reduced tone and cyanotic episodes.
Aspirin may result in a possible risk of Reyes syndrome; regular use may impair platelet
function, as well as producing a hypoprothrombinaemia, especially if the infant is low in
Vitamin K stores.

43) Theme : Genital disorders


A. Aarskog syndrome
B. Fragile X
C. Gonadal dysgenesis
D. Hermaphroditism
E. Polycystic ovaries

F. Testicular feminisation syndrome


G. XO syndrome
H. XXX syndrome
I. XXY syndrome
J. XYY syndrome

Select the most appropriate diagnosis from the list above that best describes the following cases
1) A young child is noted to have Cryptorchidism and a shawl scrotum.
A. Aarskog syndrome
Note:
describes a boy with Aarskog syndrome. Features include mild moderate learning difficulties, short
stature, facial dysmorphic features include hypertelorism anteverted nostrils and a webbed neck. Genital
abnormalities include shawl odW scrotum and cryptorchidism 8 (hidden or obscure testis).
2) A 12 year old boy is noted to have a long thin face, micropenis, hypospadias and has learning
difficultie
J. XYY syndrome s.
Note:
relates to a child with XYY syndrome. Individuals are usually tall and have learning difficulties and
abhorrent pqrst behaviour.
3) A child is noted to have macrocephaly and macroorchidism.
B. Fragile X
Note:
describes a boy with Fragile X syndrome. Features include moderate learning difficulties, speech delay
and autistic features. They have large heads and prominent ears as well as testicular enlargement.

44) Theme : ABDOMINAL PAIN


A. Peritonitis
B. Appendicitis
C. Constipation
D. Urinary tract infection
E. Mesenteric lymphadenitis

F. Migraine
G. Intussusception
H. Henoch-Schonlein vasculitis
I. Psychosomatic
J. Shigella dysentry

For the following patients with abdominal pain, choose the most likely diagnosis.
1) A 7 year old girl who has felt anorexic since the previous evening and has vomited 3 times since. She
has abdominal pain, which was made worse by the bumpy car journey to hospital.
B. Appendicitis
Note:
A typical description of appendicitis. In the younger child/ baby there is often a later presentation with
perforation of the appendix being relatively more common, as these children are not able to articulate
their pain and localization of pain is more difficult to discern with examination.
2) A 4 year old boy who has a sore throat, mild fever and peri-umbilical abdominal pain.
E. Mesenteric lymphadenitis

Pediatrics 25

Note:
Preceding sore throat and periumbilical pain are typical of mesenteric lymphadenitis in a relatively well
child.
3) A 9 year old boy has intermittent severe generalized colicky abdominal pain which is getting worse. He
has started to vomit over the last 24 hours. He has a history of recurrent abdominal pain. Examination
reveals a mass in the left iliac fossa.
C. Constipation
Note:
This is a description of chronic severe constipation, which in extreme cases can cause obstruction. There
is faecal loading implying the chronicity. When it is this severe it is important to rule out other pathology
and consider the possibility that is a manifestation of child abuse

45) Theme : PRECOCIOUS PUBERTY


A. Craniopharyngioma
B. Exaggerated adrenarche
C. McCune-Albright syndrome
D. Hypothalamic hamartoma
E. Thelarche variant

F. Neurofibromatosis type 1
G. Granulosa-theca cell tumour
H. Hypothyroidism
I. Premature thelarche
J. Congenital adrenal hyperplasia

Which is the most likely diagnosis in the following cases:


1) A 4 year old girl present s with vaginal bleeding and caf au-lait pigmentation which stops at the
midline. Subsequent investigations demonstrate gonadotrophin-independent puberty
C. McCune-Albright syndrome
Note:
McCune Albright syndrome is caused by an activating mutation of the GS alpha subunit. This results in
autonomous secretion of certain hormones e.g. thyroxine, cortisol, oestradiol. There may be associated
skin pigmentation and fibrous dysplasia.
2) A 3 year old girl presents with pubic hair development, cliteromegaly and acnes. There was an
acceleration in her height velocity and no symptoms of oestrogen excess. Neurological examination was
normal.
J. Congenital adrenal hyperplasia
Note:
Virilising Congenital adrenal hyperplasia (CAH) typically presents with symptoms of androgen excess.
There is often behavioural disturbance with mood swings. Diagnosis is confirmed by a raised 17
hydroxyprogesterone. Cliteromegaly and a sustained increase in height velocity does not occur in
adrenarche.
3) A 2year old girl presents with isolated bilateral breast development. Examination is otherwise normal
as is her height velocity. Subsequent investigation demonstrates undetectable oestradiol levels and a
prepubertal response to an LHRH test.
I. Premature thelarche
Note:
There may be a slight FSH predominance on the basal gonadotrophin levels. Premature thelarche is self
limiting and thought to be the result of the body being exquisitely sensitive to low levels of oestradiol.

Pediatrics 26

46) Theme : Causes of vomiting


A. Gastro-oesophageal reflux
B. Intussusception
C. Meconium ileus
D. Meconium plug syndrome
E. Meckel's Diverticulum

F. Mid gut volvulus


G. Necrotising enterocolitis
H. Peptic ulcer
I. Pyloric stenosis
J. Tracheo-oesophageal fistula with atresia

Select the most appropriate option from the above list that would account for the following
presentations:
1) 5 week old breast fed baby is admitted with a 2 week history of vomiting and dehydration. On
examination an olive sized mass is palpable.
I. Pyloric stenosis
Note:
Pyloric stenosis is most prevalent amongst first born male and age of onset is usually between 3-6 weeks
of age. Babies present with projectile vomiting and examination may reveal visible peristaltic waves and
an olive sized mass may be palpable in the upper epigastrium.
2) A 3 month old baby boy is admitted with episodic screaming. On examination he is mottled and has a
delayed capillary refill time. Palpation of the abdomen reveals a sausage shaped mass.
B. Intussusception
Note:
Intussusception a segment of bowel becomes telescoped in to an adjacent of bowel often in areas
where the bowel is aperistaltic, for example Peyer's patches, Meckel's diverticulum, tumour or an area of
oedema secondary to Henoch Schonlein purpura. The child presents with pain, vomiting and signs of
shock. A mass may be palpable in the right upper quadrant.
3) A baby with cystic fibrosis develops vomiting and abdominal distension. A ground glass appearance is
seen on x-ray.
C. Meconium ileus
Note:
Meconium ileus syndrome is the commonest form of small bowel obstruction and the baby presents with
distension, vomiting and constipation. An x-ray may reveal a ground glass appearance.

47) Theme : Gross motor development


A. 1 month
B. 2 months
C. 4 months
D. 6 months
E. 8 months

F. 10 months
G. 1 year
H. 18 months
I. 2 years
J. 3 years

For each description of a child's gross motor development choose, from the list of options, the
highest development age the child is most likely to have achieved.
1) A baby has good head control with no head lag when pulled from supine to sitting.
C. 4 months
2) A baby can rise independently and stands momentarily alone. Is starting to take 1-2 steps.
G. 1 year
3) A child who is walking and runs stiffly can climb stairs if hand is held.
I. 2 years

Comments:
This question tests your knowledge of the development stages of gross motor skills.
1) At 3 months when placed prone an infant can lift his head with his arms extended.
2) By 4 months head lag is lost and this is evident when raised from supine to the sitting position.
3) At 5 months the baby starts to roll over.
4) At 6 months the baby is starting to sit up with support for example cushions.

Pediatrics 27

5)
6)
7)
8)
9)

Sitting unsupported is usually seen by 6-7 months and


At 9 months the baby can start to crawl.
Cruising is usually achieved by about 10 months.
At 12 months the baby can rise independently and often starts to take a few steps alone.
At 18 months the toddler can run although this is usually in a tiff fashion. He or she can ascend
stairs with assistance.
10) At 20 months the child can climb the stairs holding on to a rail.
11) At 3 the child can ascend stairs in an adult fashion using alternating feet per step, however is
unable to descend the stairs in such a fashion until 4 years of age.
48) Theme : SHORT STATURE
A. Constitutional delay of growth
B. Growth hormone deficiency
C. Psychosocial deprivation
D. Familial short stature
E. Skeletal dysplasia

F. Hypothyroidism
G. Craniopharyngioma
H. Turners syndrome
I. Cushings syndrome

Which is the most likely diagnosis in the following cases?


1) A previously abused 4 year old child presents with growth failure over the last 12 months. She was
placed in foster care for the last 18 months and is said to much happier. Her height velocity has slowed
down and she has gained weight. She suffers with constipation and her teacher has noticed her attention
span has got worse recently.
F. Hypothyroidism
Note:
Height velocity will improve once abuse is removed. Constipation and poor attention span are common
symptoms of hypothyroidism in children.
2) A 7 year old boy was referred to clinic for investigation of short stature. He is well and apart from
occasional headaches has no other complaints. His height is below the 0.4 th centile and within the
target centile range calculated from his parents heights. Previous height measurements from his GP
show no evidence of a reduction in height velocity.
D. Familial short stature
Note:
Short stature with no evidence of growth failure and a height within the target centile range indicate
familial short stature which requires no further investigation.
3) A 3 year old girl presents with stature well below the 0.4 th centile. She had a past history of pedal
oedema at birth and uncomplicated repaired coarctation of the aorta. She has been treated for
constipation in the past but this has resolved, other wise she is very well. An insulin tolerance test
demonstrates a peak growth hormone level of 25 mU/l.
H. Turners syndrome
Note:
The insulin tolerance test should only be performed if there is growth data that suggests growth failure
and other causes of short stature have been ruled out. GH deficiency is not suggested here. She is very
short and also well i.e. other pathology unlikely. A previous history of coarctation and pedal oedema
suggests Turners syndrome.

49) Theme : Childhood malignancies


A. Burkitt's nasopharyngeal carcinoma
B. Ewing sarcoma
C. Hepatoblastoma
D. Hodgkin's lymphoma
E. Melanoma

F. Nesidioblastosis
G. Neuroblastoma
H. Teratoma
I. Wilm's tumour
J. Xeroderma pigmentosum

Select the most appropriate oncological diagnosis from the above list of options to explain the
following scenarios.

Pediatrics 28

1) A baby is born with a solitary mass over the right buttock.


H. Teratoma
2) A young child presents with fevers and a mass in the femur is noted and a diagnosis of malignancy is
made. This malignancy is 30 times more common in white children compared to black children.
B. Ewing sarcoma
3) A child of Nigerian parents presents with a unilateral tender cervical node. A diagnosis of malignancy
is made with this malignancy being 7 times more common in Black Children compared with White
children.
A. Burkitt's nasopharyngeal carcinoma

Comments:
Item 1 relates to a baby born with a sacrococcygeal teratoma. A solid tumour found in new-borns with
girls being affected more than boys. It is a mixed germ cell tumour and may be associated with
gastrointestinal or genital urinary symptoms. Ewing sarcoma is a tumour of the bone, usually found in
long bones particularly the femur. It may present with pain, fever and tenderness and sometimes
pathological fractures. There is a poor prognosis. It is 30 times more common in white children than black
children. This genetic predisposition is contrary to the Burkitt's nasopharyngeal carcinoma which is more
common in black children than white (by 7 fold). This tumour has been associated with the Epstein-Barr
virus and children present with sore throats, unilateral tenderness, cervical lymphnodes, trismus and
weight loss.

50) Theme : Cranial Nerves


A. Abducens
B. Auditory
C. Facial
D. Glossopharyngeal
E. Oculomotor

F. Olfactory
G. Optic
H. Trochlear
I. Trigeminal
J. Vagus

Which of the above cranial nerves is affected in the following cases?


1) A 3 year old girl recovering from a pneumococcal meningitis complains of a painful right eye. The
pupil is enlarged and the eyes deviated inferiorly and laterally. There is a right sided ptosis.
E. Oculomotor
2) A 12 year old boy complains of a numb feeling to the left side of his face. He is unable to whistle or
show is teeth on that side.
C. Facial
3) A 15 year old girl on treatment for Nephrotic syndrome presents with a vesicular rash involving the
right conjunctiva and right forehead, which is very tender to touch.
I. Trigeminal

Comments:
The oculomotor nerve innovates the superior inferior and medial rectus as well as the inferior oblique
and the levator palpebrae muscles. Complete paralysis of the oculomotor nerve causes ptosis, dilatation
of the pupil and displacement of the eye outwards and downwards.
An upper motor neurone facial nerve palsy is diagnosed by decreased voluntary movement of the lower
face with flattening of the nasal folds, where as a lower motor neurone lesion tends to involve the upper
and lower facial muscles.
The Trigeminal nerve sensory distribution to the face is divided in to ophthalmic, maxillary and the
mandibular routes and motor function may be tested by examining the masseter and temporalis muscles.

Pediatrics 29

51) Theme : Causes of Headaches


A. Benign intra-cranial hypertension
B. Cerebral tumour
C. Cluster headache
D. Encephalitis
E. Pseudotumour Cerebri

F. Meningitis
G. Migraine
H. Subdural haemorrhage
I. Sub arachnoid haemorrhage
J. Tension headache

Select the most appropriate diagnosis from the above list that would explain the following cases.
1) A 6 year old girl with a history of Still's disease presents with a 2 month history of headache and
diplopia She is admitted to hospital. Investigations carried out include a normal CT scan, normal CSF
profile and normal opening pressures.
E. Pseudotumour Cerebri
2) A 4 month old infant presents with poor feeding and irritability. On examination he has a torticollis and
his head circumference has increased form the 50th 98th centile since his last measurement 6 weeks
ago.

3) A 16 year old obese girl with a long history of headaches is found to have papilloedema. The rest of
the neurological examination is normal as is the CT scan.
A. Benign intra-cranial hypertension

Comments:
Intra-cranial tumours- brain tumours are second to leukaemia as the most prevalent malignancy in
childhood. Children present with either signs of increased intra-cranial pressure or with focal neurological
signs. Infratentorial tumours are more common in the paediatric age group than supratentorial tumours
and the MRI scan is the best radiological tool for delineating brain tumours. Pseudotumour cerebri is a
clinical syndrome mimicking brain tumours and characterised by normal CSF pressure and cell count and
a normal structure to the brain. Causes of which are multiple for example metabolic disorders, infections,
drugs, haematological disorders. Treatment focuses towards the underlying aetiology and pseudo tumour
cerebri is mainly self limiting. Benign intracranial hypertension is associated with pregnancy as well as
drugs such as the oral contraceptive, tetracyclines and rarely Growth Hormone therapy. It is more
common in association with obesity. LP usually reveals high opening pressures and treatment includes
removal of any precipitant with possible serial LP and removal of CSF.

52) Theme : Causes of abdominal pain


A. Acute appendicitis
B. Inflammatory bowel disease
C. Irritable bowel syndrome
D. Meckel's diverticulum
E. Muscle strain

F. Ovarian cysts
G. Pelvic inflammatory disease
H. Psoas haematoma
I. Pyelonephritis
J. Ureteric calculus

Select the most appropriate diagnosis from the above list for the following presentations
1) A 13 year old boy with umbilical pain for the last 4 days presents with anorexia, nausea and has not
passed a bowel motion for 24 hours.
A. Acute appendicitis
Note:
The pain of acute appendicitis starts off as a peri-umbilical pain which then locates to the right iliac fossa
and is some times referred to the back. It is sharp and steady; it results in nausea, emesis and local
tenderness.
2) A 10 year old boy with lower abdominal pain for the last 10 days presents with a history of passing 6-8
loose stools. Temperature is 38.8C. He is tender in the right lower quadrant and has an anal fistula.
B. Inflammatory bowel disease
Note:

Pediatrics 30

Inflammatory bowel disease results in recurrent lower abdominal pain, often radiating to the back. The
pain is dull and crampy in nature and is associated with fever, weight loss and tenesmus.
3) A 14 year old haemophiliac patient presents with lower abdominal pain and a limp. Attempts to
straighten his leg results in excruciating pain
H. Psoas haematoma
Note:
Bleed into the Psoas muscle which may be due to coagulopathy or secondary to sepsis (DIC) results in
back pain radiating to the hip with painful flexion at the hip joint.

53) Theme : NORMAL DEVELOPMENT


A. 4 weeks
B. 2 months
C. 7 months
D. 10 months
E. 12 months

F. 18 months
G. 2 years
H. 3 years
I. 4 years
J. 5 years

At what age would you expect the average child to achieve the following milestones:1) Show casting, able to pick up small objects with a neat pincer grasp and have developed concept of
object permanence.
E. 12 months
2) Able to skip, draw a triangle from copy, name the heavier of 2 weights, count up to ten and ask
questions about meaning of words.
J. 5 years
3) Able to identify colours
H. 3 years

Comments:
It is important to be familiar with the developmental milestones, particularly up to the age of 3 years.
Milestones are grouped under 4 headings:

Gross motor
Fine motor
Hearing and language
social

You should know the age range for each milestone. Developmental milestones are in Illingworth: 'The
Normal Child'.
54) Theme : Chemotherapy
A. 6-Mercaptopurine
B. Actomyosin D
C. Asparaginase
D. Bleomycin
E. Cisplatin

F. Cyclophosphamide
G. Cytosine arabinoside
H. Doxorubicin
I. Methotrexate
J. Vincristine

Select the most appropriate chemotherapeutic agent from the above list that corresponds with
the following actions and side effects:
1) An alkylating agent resulting in haemorrhagic cystitis.
F. Cyclophosphamide
2) A drug inhibiting protein synthesis may result in a dose related Pancreatitis.
C. Asparaginas

Pediatrics 31

3) An anthracycline with a serious risk of cardio-toxicity.


H. Doxorubicin

Comments:
All cytotoxic agents have adverse effects. General side-effects include nausea,
vomiting and bone marrow suppression, alopecia and stomatitis. More specific sideeffects are listed in the questions above. Alkylating agents such as Cyclophosphamide
may cause a haemorrhagic cystitis. Asparaginase results in a dose related Pancreatitis.
The Anthracyclines e.g. (Doxorubicin and Daunorubicin) are cardiotoxic, which is
often very difficult to detect. Monitoring with echocardiograms is advised.
Vincristine is an alkaloid agent and results in sensory motor neuropathy with longterm use. It also has an affect on the autonomic system resulting in severe constipation
and paralytic ileus. It may also result in sensory changes with parathesis progressing to
loss of tendon reflexes.
Bleomycin is the main drug resulting in lung damage and occurs in up to 10% of
patients. The damage is dose related.
55) Theme : SYNDROMES
A. Down's syndrome
B. Turner's syndrome
C. Noonan's syndrome
D. Klinefelter's syndrome
E. Treacher-Collins syndrome

F. Marfan's syndrome
G. Sotos syndrome
H. Klippel-feil syndrome
I. Laurence Moon Biedl syndrome
J. MCune Albright syndrome

Which is the most applicable diagnosis for the following children?


1) A boy with learning difficulties and short stature is found to have pulmonary stenosis when investigated
for a murmur. He has some dysmorphic features including wide carrying angle of the arms.
C. Noonan's syndrome
Note:
Children with Noonans syndrome have short stature and may have delayed puberty. There is also
associated short sightedness. Pulmonary stenosis is the most frequent congenital heart lesion.
2) Boy is seen in cardiology clinic for investigation of a murmur heard on routine check. His father died at
an early age from aortic dissection. He is noted to be above the 98th centile in height for his age.
F. Marfan's syndrome
Note:
Marfans syndrome is also associated with a high arched palate, arachnodactly and a predisposition to
pneumothoraces.
3) A child with learning difficulties was noticed to be long at birth. He has a large head, downslanting
eyes, bossed forehead and prominent jaw. He is clumsy and previously been investigated for hypotonia.
G. Sotos syndrome
Note:
This is a typical description of Sotos syndrome which is one of the overgrowth syndromes.

Pediatrics 32

56) Theme : Connective tissue disease


A. Ankylosing spondylitis
B. Dermatomyositis
C. Ehlers Danlos syndrome
D. Henoch Schonlein purpura
E. Juvenile chronic arthritis

F. Kawasaki disease
G. Osteogenesis imperfecta
H. Rheumatic fever
I. Scleroderma
J. Systemic lupus erythematosus

Select one of the above diagnoses that would explain the following presentations:
1) A 12 year old girl presents with oesophageal dysmotility, interstitial pneumonitis and has cardiac
conduction abnormalities (arrhythmias).
I. Scleroderma
Note:
describes a child with scleroderma, a chronic fibrotic disturbance which tends to be relapsing and
remitting. It affects the GI tract, heart lungs and kidneys.
2) A 5 year old presents with conjunctivitis fever and desquamation of the periphery.
F. Kawasaki diseas
Note:
Kawasaki's disease : is also known as mucocutaneous lymph node syndrome. It is a clinical
diagnosis made by a history of fever for 5 days, bilateral conjunctivitis, mucosal involvement and
peripheral changes including erythema and desquamation. Cardiac involvement is present in
approximately 30% of cases and this is a serious complication.
3) A 10 year old girl presents with weight loss, peri-orbital oedema, weakness and is noted to have a
photosensitive rash.
B. Dermatomyositis
Note:
relates to the diagnosis of Dermatomyositis, a multi system disease where individuals have a
violaceous heliotrope rash around the eyes, Guttrons patches on the knuckles and inflammation of
muscles and joints resulting in weakness in muscles and arthritis. The condition is often associated with
underlying malignancy.

57) Theme : Causes of abdominal pain


A. Acute appendicitis
B. Inflammatory bowel disease
C. Irritable bowel syndrome
D. Meckel's diverticulum
E. Muscle strain

F. Ovarian cysts
G. Pelvic inflammatory disease
H. Psoas haematoma
I. Pyelonephritis
J. Ureteric calculus

Select the most appropriate diagnosis from the above list for the following presentations
1) A 7 year old boy presents with a 2 weeks history of flank pain and rigors, intermittent fevers.
I. Pyelonephritis
Note:
Pyelonephritis - an infection of the renal tract causes acute sudden pain in the lower abdomen and back.
This is associated with urinary frequency and dysuria, costochondral tenderness and fever.
2) A 13 year old girl presents with recurrent abdominal pain and haematuria. She is apyrexial
J. Ureteric calculus
Note:
Urolithiasis or urinary tract calculi present with haematuria (macro or microscopic) abdominal pain,
dysuria and voiding abnormalities.
3) An 8 year old girl with a history of recurrent central abdominal pain presents with pallor and an Iron
deficiency anaemia.
D. Meckel's diverticulum
Note:
Meckel's diverticulum results in peri-umbilical sharp pain associated with haematochezia

Pediatrics 33

Haematochezia (bright red blood per rectum)


58) Theme : Causes of headache and fever
A. Bacterial meningitis
B. Congenital rubella
C. Congenital toxoplasmosis
D. Herpes simplex encephalitis
E. Leukaemic infiltration

F. Meningococcal meningitis
G. Pneumococcal meningitis
H. Status epilepticus
I. Tuberculous meningitis
J. Varicella cerebellitis

Select the most likely diagnosis from the above list that would account for the following
presentations
1) A child is admitted with alternating levels of consciousness and a high temperature. EEG shows
periodic, lateralised epileptiform discharges.
D. Herpes simplex encephalitis
2) A baby is born at term weighing 2.5kilos. He is noted to have microcephaly. An ultrasound scan of his
head reveals calcification.
C. Congenital toxoplasmosis
3) A baby of 6 months has a full septic screen. His CSF shows 10 white cells predominantly
lymphocytes, high protein and low glucose.
I. Tuberculous meningitis
Comments:
Examination of the cerebral spinal fluid is essential in confirming the diagnosis of conditions such as
meningitis, encephalitis and sub arachnoid haemorrhage. The main contraindication for performing a
lumbar puncture is any condition that may lead to Coning or Herniation of intra-cranial contents.
Therefore fundoscopy to exclude the presence of papilloedema is essential. Routine CSF study includes
red cell count, white cell count, glucose protein and bacterial cultures.
In meningitis typical CSF findings include IA neutrophilic pleocytosis with an elevated protein and
reduced glucose. Gram stains reveal organisms in approximately 80% of cases of bacterial meningitis.
In encephalitis CSF is typically : normal. In Tuberculous meningitis I the cell count is elevated
and the predominance of lymphocytes. Glucose is reduced (1/3 the serum) however the protein
concentration is elevated (0.5-6 GG) (normal protein g/l 0.2-0.4). With Leukaemic infiltration of the
central nervous system atypical I Leukaemic cells may be present in the CSF. Further cytopathology
studies may be indicated.
Congenital toxoplasmosis :affected infants may have fever, rash, petechiae, hepatosplenomegaly.
They may have dysmorphic features such as microcephaly or hydrocephalus, microphthalmia and
chorioretinitis. Cerebral calcification is common. Prognosis is guarded as affected patients may have
persistent neurological disability. (Brain Calcification in CMV infection :periventricular , in toxoplasmosis
: widespread No calcification in rubella)

59) Theme : Cranial Nerves


A. Abducens
B. Auditory
C. Facial
D. Glossopharyngeal
E. Oculomotor

F. Olfactory
G. Optic
H. Trochlear
I. Trigeminal
J. Vagus

Which of the above cranial nerves is affected in the following cases?


1) A 16 year old female who is pregnant complains of headaches and has blurring of the disc margin on
fundoscopy.
G. Optic
2) A baby is born with an ipsilateral diffuse birth mark over the right side of his face. He develops
seizures in the second week of life.
I. Trigeminal

Pediatrics 34

3) A 15 year old boy recovering from a head injury is unable to abduct his right eye beyond the mid-line.
A. Abducens

Comments:
The first case has probable benign intracranial hypertension causing papilloedema. The optic nerve is
examined by fundoscopy as well as visual acuity and visual fields. The Trigeminal nerve sensory
distribution to the face is divided in to ophthalmic, maxillary and the mandibular routes and motor function
may be tested by examining the masseter and temporalis muscles. The abducens nerve innovates
bilateral rectus muscle of the eye therefore paralysis causes medial; deviation of the eye and inability to
abduct the eye beyond the mid-line.
60) Theme: Childhood syndromes.
A. Alport syndrome
B. Downs syndrome
C. Duchenne muscular dystrophy
D. Edwards syndrome
E. Gauchers syndrome

F. Klinefelters syndrome
G. Pierre robin syndrome
H. Sturge webber syndrome
I. Treacher collins syndrome
J. Turners syndrome

For each patient below choose from the list above the single most likely diagnosis from the list
above. Each option may be chosen more than once or not at all.
1) A neonate is seen with short chin cleft palate and feeding and respiratory difficulties.
G. Pierre robin syndrome
Note:
Pierre Robin syndrome. Results in neonatal difficulty in feeding, micrognathia (short chin due to
hypoplasia of mandible) cleft palate and eye abnormality. These babies should be nursed prone to
prevent obstructive apnoea.
2) A 9 month old baby with low short receding chin low set ears, deafness and notching of lower eye lids.
I. Treacher collins syndrome
Note:
Treacher Collins syndrome. Due developmental abnormalities of first and second pharyngeal pouch.
Low set ears (80%), deafness 40%, under developed mandible.

3) A 16 year old girl presents with primary amenorrhoea, short stature, and radio-femoral delay.
J. Turners syndrome
Note:
Turners syndrome. 45XO Chromosomal abnormality. Associated with short stature (<130 cm), primary
amenorrhoea, rudimentary gonads, wide carrying angle, webbing of the neck cardiac abnormalities.

4) A 6 week old baby with low set ears, receding chin umbilical hernia and rocker bottom feet.
D. Edwards syndrome
Note:
Edwards syndrome (Trisomy 18). Rigid baby, flexion of limbs, mental retardation. Mean survival less
than 10 months.
5) A 5 year old boy presents with waddling gait and positive Gowers sign.
C. Duchenne muscular dystrophy
Note:
Duchennes muscular dystrophy X-linked recessive .

Pediatrics 35

61) Theme : Childhood Breathlessness


A. Asthma
B. Bronchiolitis
C. Bronchiectasis
D. Broncho pulmonary dysplasia
E. Cardiac failure

F. Laryngomalacia
G. Tracheal haemangioma
H. Tracheal oesophageal fistula
I. Vitamin D deficiency
J. Vocal cord palsy

Select the most appropriate choice from the above list that is the most appropriate diagnosis in
the following cases
1) A premature baby presents with a cough and cyanotic episodes during feeds.
H. Tracheal oesophageal fistula
2) A girl with hypogammaglobulinaemia develops haemoptysis and a purulent sputum.
C. Bronchiectasis
3) A baby is born by a ventouse delivery following failed forceps. She is born in poor condition and
stridor is noted shortly after birth.
J. Vocal cord palsy

Comments:
Item 1describes a baby with tracheo-oesophageal fistula, which typically presents with a cough and
choking cyanotic episodes during feeds. Bronchiectasis typically produces clubbing and haemoptysis as
well as persistent crepitations in the chest. This process may follow a severe pneumonia for example
pertussis, measles, TB. It can also complicate immune-deficiency states and Cystic fibrosis.
Kartagener's syndrome is a triad of 1) bronchiectasis 2) sinusitis 3) dextrocardia. The features result
from abnormal ciliary function. Vocal cord paralysis is the answer to Item 6. Stridor from birth may occur
as a result of birth trauma or an abnormality of the central nervous system, such as hydrocephalus.
62) Theme : Paediatric endocrinology
A. 3 Beta hydroxysteroid dehydrogenase deficiency
B. 17 Hydroxylase deficiency
C. 21 Hydroxylase deficiency
D. Adrenal hyperplasia
E. Craniopharyngioma

F. Hypoadrenalism
G. Phaeochromocytoma
H. Panhypopituitarism
I. Polyglandular endocrinopathy
J. Primary aldosteronism

Select the most appropriate diagnosis from the above options to explain the following
presentations:
1) A child is admitted to ITU with meningococcaemia. On arrival hyponatraemia and hyperkalaemia is
revealed. Urinary electrolytes reveal high levels of Sodium and Chloride and low levels of Potassium.
F. Hypoadrenalism
Note:
describes a child with meningococcaemia complicated by hypoadrenalism of Waterhouse-Friderichsen
syndrome. This is due to haemorrhage into the adrenal glands. If the condition is unrecognised adrenal
crisis may result and therefore treatment must be immediate and vigorous (Hydrocortisone, IV fluids and
Glucose).
2) A child attending diabetic clinic complains of feeling tired. She is found to have thin hair, patches of
vitiligo and the tongue is coated in white. Blood tests reveal low Calcium and high Phosphate.
I. Polyglandular endocrinopathy
Note:
describes a child with Polyglandular endocrinopathy. In this case a child with diabetes, vitiligo,
Addison's disease, hypoparathyroidism and chronic mucocutaneous Candidiasis (type 1 Polyglandular
endocrinopathy.) Type 2 Polyglandular endocrinopathy is diagnosed if there is history of hyperthyroid
disease or diabetes plus Addison's disease.
3) A baby is born at term. Birth weight is on the 50th centile. Within the first few weeks hypoglycaemia is
diagnosed. The growth of the baby is monitored closely and By 9 months of age parameters have fallen
to the second centile.
H. Panhypopituitarism

Pediatrics 36

Note:
The last question describes a baby with growth hormone deficiency. From the above option
Panhypopituitarism is the answer. Babies with growth hormone deficiency present with episodes of
hypoglycaemia and may present as an emergency with apnoeic episodes or cyanotic episodes. They are
usually normal in birth size; however by 1 year of age growth has tailed off. In Panhypopituitarism the
deficiency may be of growth hormone only or of multiple hormones and therefore laboratory
investigations should include thyroid function tests, cortisol and ACTH, gonadotrophin's and prolactin as
well as growth hormone.

63) Theme : WEAKNESS IN THE LOWER LIMBS


A. Guillain-Barre syndrome
B. Transverse Myelitis
C. Multiple sclerosis
D. Chronic fatigue syndrome
E. Spinal tumour

F. Dermatomyositis
G. Congenital myopathy
H. Duchenne muscular dystrophy
I. Fascioscapulohumeral dystrophy
J. Spinal abscess

For these children with lower limb weakness select the most likely diagnosis
1) A 4 year old boy is brought to clinic by his mother who feels he has weak legs and is finding climbing
the stairs difficult. He walked independently at 20 months and has received speech therapy for a mild
speech delay. On examination his lower limb reflexes are normal, his muscles are well developed distally
although weak proximally.
H. Duchenne muscular dystrophy
Note:
DMD occurs in boys(X-linked recessive) and presents with delayed walking i.e.> 18months. Speech
delay, cardiomyopathy, scoliosis and calf pseudohypertrophy are also features.
2) A 12 year old girl presents with a 5 day history of progressive weakness in her legs and is now unable
to walk. She has been passing urine frequently and with difficulty. On examination there is power with
gravity eliminated in her legs, very brisk ankle and knee jerks and upgoing plantar responses. Her
bladder is palpable up to the umbilicus. Sensation is reduced to touch below T5, vibration and position
sense are preserved 3 weeks previously she had a presumed viral illness. MRI brain is normal but shows
swelling and increased signal over the spinal cord T5-7 on T1- weighted images. Lumbar puncture shows
33 lymphocytes and monocytes per microlitre and protein 570 mg/L in the CSF.
B. Transverse Myelitis
Note:
The features are suggestive of a lower motor neurone lesion which could also be caused by a spinal
tumour / abscess however the MRI would demonstrate these latter pathologies. The CSF lymphocytosis
and increased protein may be seen in transverse myelitis. Transverse myelitis has unknown aetiology but
has been postulated to have a viral or mycoplasma origin.
3) A 14 year old girl has been off school for 4 months with headaches, tiredness and weakness of her
legs. In the initial weeks of her illness she had a low-grade fever, sore throat and cervical
lymphadenopathy. A blood count showed atypical lymphocytes and liver function tests show a raised
AST. These were re-checked 1 month later when they had normalised and thyroid function, urine culture
urea & electrolytes were also normal. It is difficult to get her full co-operation for examination but there
are no objective signs of weakness and reflexes are all present, symmetrical with downgoing plantar
responses.
D. Chronic fatigue syndrome
Note:
The previous viral illness and subsequent lethargy with no consistent abnormal investigations suggest
chronic fatigue syndrome.

Pediatrics 37

64) Theme : FRACTURES IN CHILDREN


A. Accidental fracture
B. Child physical abuse
C. Osteoporosis
D. Rickets
E. Metabolic bone disease of prematurity

F. Hypoparathyroidism
G. Hypophosphatemic rickets
H. Pseudohypoparathyroidism
I. Osteogenesis imperfecta
J. Osteopetrosis

For each patient select the most likely diagnosis.


1) A 15 month girl is in foster care following child protection procedures when she presented with a spiral
fracture at the age of 11 months. Her mother is single, poorly supported and had been suffering from
postnatal depression at the time. There had been no explanation offered for the original injury. She now
presents with a greenstick fracture of the femur following a trivial fall in the care of the experienced fostermother who has been known to social services for many years. She is just walking independently. Skull
X-ray shows several Wormian bones.
I. Osteogenesis imperfecta
Note:
Wormian bones are indicative of osteogenesis impefecta. This condition is an important differential of
NAI.

2) An 11 month old girl is brought to casualty refusing to move her left arm. An x-ray demonstrates a
spiral fracture of the humerus. The mother says that 30 minutes previously she lifted her daughter out of
the bath and lost grip of her right hand and her body momentarily twisted round her left arm.
A. Accidental fracture
Note:
There is no delay in presentation and the mechanism of injury fit the description of the episode.
3) A 10 year old girl presents with back pain. A spinal X ray shows collapsed lumbar vertebrae at L3 and
L4 and generalized osteopenia. She has a history of 3 previous limb fractures after relatively trivial falls.
She is otherwise healthy and well-grown for her age.
I. Osteogenesis imperfecta
Note:
Collapsed vertebra and osteopenia are uncommon in children. Coupled with low trauma fractures and no
dysmorphism, type 1 osteogenesis imperfecta is likely.

65) Theme : FRACTURES IN CHILDREN


A. Accidental fracture
B. Child physical abuse
C. Osteoporosis
D. Rickets
E. Metabolic bone disease of prematurity

F. Hypoparathyroidism
G. Hypophosphatemic rickets
H. Pseudohypoparathyroidism
I. Osteogenesis imperfecta
J. Osteopetrosis

For each patient select the most likely diagnosis.


1) A toddler of 3 years presents to the accident and emergency department with swelling of his leg and is
found to have a spiral fracture of the tibia. His mother reports that he had tripped and fallen the previous
day but she had not noticed any sign of injury at the time. She is a single parent with little family support.
The child is not on the child protection register.
B. Child physical abuse
Note:
It is imperative to determine whether the mechanism of injury is compatible with the history. There is
delay in presentation and also spiral fracture indicates twisting injury rather than falling over.
2) An infant is admitted with symptoms and signs of respiratory infection and is found to have several
posterior rib fractures on chest radiograph. He was born prematurely at 37 weeks gestation and was
observed overnight on the special care baby unit for tachypnoea which settled by the following day. On
assessment it is also apparent that his head circumference has increased at an excessive rate and has
crossed 3 centiles since birth.
B. Child physical abuse
Note:

Pediatrics 38

Posterior rib fractures are highly suggestive of non accidental injury (NAI). The excessive head growth is
likely to be secondary to a previous shaking injury which has resulted in an intracerebral bleed and
hydrocephalus.
3) A 4 month baby girl is presented with swelling of her right arm and is found to have a spiral fracture of
the humerus. She had been in the care of her mothers boyfriend who reported that he had nearly
dropped her that day when reaching for her bottle and had inadvertently pulled on her arm to save her.
A. Accidental fracture
Note:
This history could be compatible with an accidental injury.

66) Theme : DELAYED MILESTONES


A. 4 weeks
B. 2 months
C. 7 months
D. 10 months
E. 12 months

F. 18 months
G. 2 years
H. 3 years
I. 5 years
J. 6 years

At what age would you refer a child for investigation if they were unable to do the following:1) Speak in intelligible short sentences
G. 2 years
2) Be reliably dry at night (i.e. primary nocturnal enuresis) whilst continent of urine and faeces by day
and otherwise well and asymptomatic.
J. 6 years
3) Be able to sit unsupported
D. 10 months

Comments:
It is important to be familiar with the developmental milestones, particularly up to the age of 3 years.
Milestones are grouped under 4 headings:

Gross motor
Fine motor
Hearing and language
Social

You should know the age range for each milestone. Developmental milestones are in Illingworth: 'The
Normal Child'.

Theme : Gastro-intestinal disorders in neonates


A. Ano-rectal atresia
B. Hirschsprungs disease
C. Infantile hypertrophic pyloric stenosis
D. Intestinal atresia
E. Intussusception
F. Meconium ileus
G. Necrotizing enterocolitis
H. Tracheo-oesophageal fistula
I. Volvulus neonatorum

Pediatrics 39

Select the most appropriate diagnosis from the list above that would explain the presentation of
the following neonates:
1) A newborn baby girl presents with gross
abdominal distension and bilious vomiting. She
has also got cystic fibrosis and her abdominal
x-ray shows distended coils of bowel, but no
fluid levels.

2) A premature infant (31-week gestation)


presents with distended and tense abdomen.
She is passing blood and mucus per rectum,
and she is also manifesting signs of sepsis.

3) A newborn baby boy presents with mild


abdominal distension and failure to pass
meconium after 24 hours. X-ray reveals dilated
loops of bowel with fluid levels.

A. Ano-rectal atresia
F. Meconium ileus

A. Ano-rectal atresia
G. Necrotizing enterocolitis

A. Ano-rectal atresia
B. Hirschsprungs disease

Comments:
One in 15 000 newborns will have a distal small bowel obstruction secondary to abnormal bulky and
viscid meconium. Ninety percent of these infants will have cystic fibrosis and the abnormal meconium is
the result of deficient intestinal secretions. This condition presents during the first days of life with gross
abdominal distension and bilious vomiting. X-ray of the abdomen shows distended coils of bowel and
typical mottled ground glass appearance. Fluid levels are scarce as the meconium is viscid. Necrotizing
enterocolitis is more common in premature infants. Mesenteric ischemia causes bacterial invasion of the
mucosa leading to sepsis. Terminal ileum, caecum and the distal colon are commonly affected. The
abdomen is distended and tense, and the infant passes blood and mucus per rectum. X-rays of the
abdomen shows distended loops of intestine and gas bubbles may be seen in the bowel wall.
Hirschsprungs disease is an absence of ganglion cells in the neural plexus of the intestinal wall. It is
more common in boys than girls. The delayed passage of meconium together with distension of
abdomen is the usual clinical presentation. A plain abdominal x-ray will demonstrate dilated loops of
bowel with fluid levels and a barium enema can be helpful when it demonstrates a cone with dilated
ganglionic proximal colon and the distal aganglionic bowel failing to distend.

Pediatrics 40

You might also like